NR 508 midterm

Pataasin ang iyong marka sa homework at exams ngayon gamit ang Quizwiz!

New over-the-counter drug ingredients must undergo the U.S. Food and Drug Administration New Drug Application process, just as prescription drugs do. 1. True 2. False

1) True

The American Dietetic Association has recommended the use of specific nutritional supplements in the following population(s): 1. 400 IU per day of vitamin D in all infants and children 2. 1,000 IU per day of vitamin D for all pregnant women 3. 60 mg per day of iron for all adults over age 50 years 4. All of the above

1. 400 IU per day of vitamin D in all infants and children

Male patients require ______________________ before and during androgen therapy. 1. A digital prostate exam 2. A Doppler exam of testicular blood flow 3. Urine analysis for proteinuria 4. Serial orthostatic blood pressures

1. A digital prostate exam

Joanne is a 60-year-old patient with an LDL of 132 and a family history of coronary artery disease. She has already tried diet changes (increased fiber and plant sterols) to lower her LDL and after 6 months her LDL is slightly higher. The next step in her treatment would be: 1. A statin 2. Niacin 3. Sterols 4. A fibric acid derivative

1. A statin

Hypertensive African Americans are typically listed as not being as responsive to which drug groups? 1. ACE inhibitors 2. Calcium channel blockers 3. Diuretics 4. Bidil (hydralazine family of medications)

1. ACE inhibitors

Increased life expectancy for patients with heart failure has been associated with the use of: 1. ACE inhibitors, especially when started early in the disease process 2. All beta blockers regardless of selectivity 3. Thiazide and loop diuretics 4. Cardiac glycosides

1. ACE inhibitors, especially when started early in the disease process

An advantage of prescribing a sublingual medication is that the medication is: 1. Absorbed rapidly 2. Excreted rapidly 3. Metabolized minimally 4. Distributed equally

1. Absorbed rapidly

Pong-tai is a 12-month-old child who is being treated with amoxicillin for acute otitis media. His parents call the clinic and say he has developed diarrhea. The appropriate action would be to: 1. Advise the parents that some diarrhea is normal with amoxicillin and recommend probiotics daily. 2. Change the antibiotic to one that is less of a gastrointestinal irritant. 3. Order stool cultures for suspected viral pathogens not treated by the amoxicillin. 4. Recommend increased fluids and fiber in his diet.

1. Advise the parents that some diarrhea is normal with amoxicillin and recommend probiotics daily.

When is aspirin (ASA) used in angina patients? 1. All angina patients should be taking ASA unless it is contraindicated for allergy or other medical reasons. 2. ASA should only be used in men. 3. ASA has no role in angina, but is useful in MI prevention. 4. The impact of ASA is best at the time of an angina attack.

1. All angina patients should be taking ASA unless it is contraindicated for allergy or other medical reasons

Premenstrual dysphoric disorder (PMDD) occurs in a fairly small number of patients. Theories of the pathology behind PMDD that are supported in research include: 1. Altered sensitivity in the serontonic system 2. Inhibition of the cyclooxygenase system 3. Fluctuations of the gonadal hormones 4. All of these are theories supported by research

1. Altered sensitivity in the serontonic system

The NP orders a thyroid panel for a patient on amiodarone. The patient tells the NP that he does not have thyroid disease and wants to know why the test is ordered. Which is a correct response? 1. Amiodarone inhibits an enzyme that is important in making thyroid hormone and can cause hypothyroidism. 2. Amiodarone damages the thyroid gland and can result in inflammation of that gland, causing hyperthyroidism. 3. Amiodarone is a broad spectrum drug with many adverse effects. Many different tests need to be done before it is given. 4. Amiodarone can cause corneal deposits in up to 25% of patients.

1. Amiodarone inhibits an enzyme that is important in making thyroid hormone and can cause hypothyroidism.

Treatment failure in patients with peptic ulcer disease associated with H. pylori may be because of: 1. Antimicrobial resistance 2. An ineffective antacid 3. Overuse of proton pump inhibitors 4. All of the above

1. Antimicrobial resistance

Jamison has been prescribed citalopram (Celexa) to treat his depression. Education regarding how quickly selective serotonin reuptake inhibitor (SSRI) antidepressants work would be: 1. Appetite and concentration improve in the first 1 to 2 weeks. 2. Sleep should improve almost immediately upon starting citalopram. 3. Full response to the SSRI may take 2 to 4 months after he reaches the full therapeutic dose. 4. His dysphoric mood will improve in 1 to 2 weeks.

1. Appetite and concentration improve in the first 1 to 2 weeks.

Beta blockers are especially helpful for patients with exertional angina who also have: 1. Arrhythmias 2. Hypothyroidism 3. Hyperlipidemia 4. Atherosclerosis

1. Arrhythmias

To reduce mortality, all patients with angina, regardless of class, should be on: 1. Aspirin 81 to 325 mg/d 2. Nitroglycerin sublingually for chest pain 3. ACE inhibitors or angiotensin receptor blockers 4. Digoxin

1. Aspirin 81 to 325 mg/d

Intimate partner violence is a serious public health problem. It should be screened for: 1. At every encounter within the health-care system 2. When a women is being seen for symptoms of depression 3. Throughout pregnancy 4. If a sexually transmitted disease is diagnosed

1. At every encounter within the health-care system

When treating suspected gonorrhea in a nonpregnant patient, the patient should be concurrently treated for chlamydia with: 1. Azithromycin 1 gram PO x 1 2. Amoxicillin 500 mg PO x 1 3. Ciprofloxacin 500 mg PO x 1 4. Penicillin G 2.4 million units IM x 1

1. Azithromycin 1 gram PO x 1

Isoniazid (INH) may induce a deficiency of which vitamin? 1. B6 2. C 3. D 4. E

1. B6

Which of the following statements about bioavailability is true? 1. Bioavailability issues are especially important for drugs with narrow therapeutic ranges or sustained-release mechanisms 2. All brands of drug have the same bioavailability 3. Drugs that are administered more than once a day have a greater bioavailability that drugs given once daily 4. Combining an active drug with an inert substance does not affect bioavailibility

1. Bioavailability issues are especially important for drugs with narrow therapeutic ranges or sustained-release mechanisms

When prescribing Adderall (amphetamine and dextroamphetamine) to adults with ADHD the nurse practitioner will need to monitor: 1. Blood pressure 2. Blood glucose levels 3. Urine ketone levels 4. Liver function

1. Blood pressure

A 19-year-old male was started on risperidone. Monitoring for risperidone includes observing for common side effects, including: 1. Bradykinesia, akathisia, and agitation 2. Excessive weight gain 3. Hypertension 4. Potentially fatal agranulocytosis

1. Bradykinesia, akathisia, and agitation

If prescribing bupropion (Zyban) for tobacco cessation, the instructions to the patient include: 1. Bupropion (Zyban) is started 1 to 2 weeks before the quit date. 2. Nicotine replacement products should not be used with bupropion. 3. If they smoke when taking bupropion they may have increased anxiety and insomnia. 4. Because they are not using bupropion as an antidepressant, they do not need to worry about increased suicide ideation when starting therapy.

1. Bupropion (Zyban) is started 1 to 2 weeks before the quit date.

Your patient calls for an appointment before going on vacation. Which medication should you ensure he has an adequate supply of before leaving to avoid life-threatening complications? 1. Carvedilol 2. Donepezil 3. Bethanechol 4. Tacrine

1. Carvedilol

Treatment for suspected gonorrhea is: 1. Ceftriaxone 250 mg IM x 1 2. Ceftriaxone 2 grams IM x 1 3. Ciprofloxacin 500 mg PO x 1 4. Doxycycline 100 mg bid x 7 days

1. Ceftriaxone 250 mg IM x 1

Pharmaceutical promotion may affect prescribing. To address the impact of pharmaceutical promotion, the following recommendations have been made by the Institute of Medicine: 1. Conflicts of interest and financial relationships should be disclosed by those providing education. 2. Providers should ban all pharmaceutical representatives from their office setting. 3. Drug samples should be used for patients who have the insurance to pay for them, to ensure the patient can afford the medication. 4. Providers should only accept low-value gifts, such as pens and pads of paper, from the pharmaceutical representative.

1. Conflicts of interest and financial relationships should be disclosed by those providing education.

Drug choices to treat angina in older adults differ from those of younger adults only in: 1. Consideration of risk factors for diseases associated with and increased in aging 2. The placement of drug therapy as a treatment choice before lifestyle changes are tried 3. The need for at least three drugs in the treatment regimen because of the complexity of angina in the older adult 4. Those with higher risk for silent myocardial infarction

1. Consideration of risk factors for diseases associated with and increased in aging

When a pharmacoeconomic analysis looks at two or more treatment alternatives that are considered equal in efficacy and compares the costs of each it is referred to as: 1. Cost-minimization analysis 2. Cost-of-illness analysis 3. Cost-effectiveness analysis 4. Cost-benefit analysis

1. Cost-minimization analysis

A 22-year-old woman receives a prescription for oral contraceptives. Education for this patient includes: 1. Counseling regarding decreasing or not smoking while taking oral contraceptives 2. Advising a monthly pregnancy test for the first 3 months she is taking the contraceptive 3. Advising that she may miss two pills in a row and not be concerned about pregnancy 4. Recommending that her next follow-up visit is in 1 year for a refill and annual exam

1. Counseling regarding decreasing or not smoking while taking oral contraceptives

Jacob has hypertension, for which a calcium channel blocker has been prescribed. This drug helps control blood pressure because it: 1. Decreases the amount of calcium inside the cell 2. Reduces stroke volume 3. Increases the activity of the Na+/K+/ATPase pump indirectly 4. Decreases heart rate

1. Decreases the amount of calcium inside the cell

Shana is receiving her first medroxyprogesterone (Depo Provera) injection. Shana will need to be monitored for: 1. Depression 2. Hypertension 3. Weight loss 4. Cataracts

1. Depression

Men who are prescribed sildenafil (Viagra) need ongoing monitoring for: 1. Development of chest pain or dizziness 2. Weight gain 3. Priapism 4. Renal function

1. Development of chest pain or dizziness

A patient's nutritional intake and laboratory results reflect hypoalbuminemia. This is critical to prescribing because: 1. Distribution of the drugs to the target tissue may be affected 2. The solubility of the drug will not match the site of absorption 3. There will be less free drug available to generate an effect. 4. Drugs bound to albumin are readily excreted by the kidneys

1. Distribution of drugs to target tissue may be affected.

You are treating a patient with a diagnosis of Alzheimer's disease. The patient's wife mentions difficulty with transportation to the clinic. Which medication is the best choice? 1. Donepezil 2. Tacrine 3. Doxazosin 4. Verapamil

1. Donepezil

Treatments for heart failure, including drug therapy, are based on the stages developed by the ACC/AHA. Stage A patients are treated with: 1. Drugs for hypertension and hyperlipidemia, if they exist 2. Lifestyle management including diet, exercise, and smoking cessation only 3. Angiotensin-converting enzyme (ACE) inhibitors to directly affect the heart failure only 4. No drugs are used in this early stage

1. Drugs for hypertension and hyperlipidemia, if they exist

Several classes of drugs have interactions with cholinergic blockers. Which of the following is true about these interactions? 1. Drugs with a narrow therapeutic range given orally may not stay in the GI tract long enough to produce an action. 2. Additive antimuscarinic effects may occur with antihistamines. 3. Cholinergic blockers may decrease the sedative effects of hypnotics. 4. Cholinergic blockers are contraindicated with antipsychotics.

1. Drugs with a narrow therapeutic range given orally may not stay in the GI tract long enough to produce an action.

An elderly person has been prescribed lactulose for treatment of chronic constipation. Monitoring with long-term treatment would include: 1. Electrolytes, including potassium and chloride 2. Bone mineral density for osteoporosis 3. Magnesium level 4. Liver function

1. Electrolytes, including potassium and chloride

Richard is 70 years old and has a history of cardiac dysrhythmias. He has been prescribed nadolol. You do his annual laboratory work and find a CrCl of 25 ml/min. What action should you take related to his nadolol? 1. Extend the dosage interval. 2. Decrease the dose by 75%. 3. Take no action because this value is expected in the older adult. 4. Schedule a serum creatinine level to validate the CrCl value.

1. Extend the dosage interval.

Clinical judgement in prescribing includes: 1. Factoring in the cost to the patient of the medication prescribed. 2. Always prescribing the newest medication available for the disease process 3. Handing out drug samples to poor patients 4. Prescribing all generic medications to cut cost

1. Factoring in the cost to the patient of the medication prescribed

Taylor is a 10-year-old child diagnosed with major depression. The appropriate first-line antidepressant for children is: 1. Fluoxetine 2. Fluvoxamine 3. Sertraline 4. Escitalopram

1. Fluoxetine

Rabi is being prescribed phenytoin for seizures. Monitoring includes assessing: 1. For phenytoin hypersensitivity syndrome 3 to 8 weeks after starting treatment 2. For pedal edema throughout therapy 3. Heart rate at each visit and consider altering therapy if heart rate is less than 60 bpm 4. For vision changes, such as red-green blindness, at least annually

1. For phenytoin hypersensitivity syndrome 3 to 8 weeks after starting treatment

What does the provider understand about the issue of "Diabetic Renal Protection" with angiotensin-converting enzyme (ACE) medications? Diabetes mellitus patients: 1. Have a reduced rate of renal progression, but still need to be discontinued when advanced renal issues present 2. Who start these medications never progress to renal nephropathy 3. With early renal dysfunction will see it reverse when on ACE medications 4. Without renal issues are the only ones who benefit from ACE protection

1. Have a reduced rate of renal progression, but still need to be discontinued when advanced renal issues present

Long-term use of proton pump inhibitors may lead to: 1. Hip fractures in at-risk persons 2. Vitamin B6 deficiency 3. Liver cancer 4. All of the above

1. Hip fractures in at-risk persons

Prior to starting antidepressants, patients should have laboratory testing to rule out: 1. Hypothyroidism 2. Anemia 3. Diabetes mellitus 4. Low estrogen levels

1. Hypothyroidism

Newborns are at risk for early vitamin K deficiency bleeding and the American Academy of Pediatrics recommends that all newborns receive: 1. IM vitamin K (phytonadione) within 24 hours of birth 2. Oral vitamin K supplementation in the first 3 weeks of life 3. Formula containing vitamin K or breast milk 4. Oral vitamin K in the first 24 hours after birth

1. IM vitamin K (phytonadione) within 24 hours of birth

Kirk sprained his ankle and is asking for pain medication for his mild-to-moderate pain. The appropriate first-line medication would be: 1. Ibuprofen (Advil) 2. Acetaminophen with hydrocodone (Vicodin) 3. Oxycodone (Oxycontin) 4. Oral morphine (Roxanol)

1. Ibuprofen (Advil)

Because of the pattern of cholesterol synthesis, reductase inhibitors are given: 1. In the evening in a single daily dose 2. Twice daily in the morning and the evening 3. With each meal and at bedtime 4. In the morning before eating

1. In the evening in a single daily dose

A low-carbohydrate, high-protein diet may: 1. Increase drug-metabolizing enzymes 2. Decrease drug absorption from the GI tract 3. Alter drug binding to plasma proteins 4. Enhance drug elimination

1. Increase drug-metabolizing enzymes

Bethanechol: 1. Increases detrusor muscle tone to empty the bladder 2. Decreases gastric acid secretion to treat peptic ulcer disease 3. Stimulates voluntary muscle tone to improve strength 4. Reduces bronchial airway constriction to treat asthma

1. Increases detrusor muscle tone to empty the bladder

The health-care delivery system itself can create barriers to adherence to a treatment regimen. Which of the following system variables creates such a barrier? 1. Increasing copayments for care 2. Unrestricted formularies for drugs, including brand names 3. Increasing the number of people who have access to care 4. Treating a wider range of disorders

1. Increasing copayments for care

Patients who are prescribed olanzapine (Zyprexa) should be monitored for: 1. Insomnia 2. Weight gain 3. Hypertension 4. Galactorrhea

1. Insomnia

Patients who are on chronic long-term proton pump inhibitor therapy require monitoring for: 1. Iron deficiency anemia, vitamin B12 and calcium deficiency 2. Folate and magnesium deficiency 3. Elevated uric acid levels leading to gout 4. Hypokalemia and hypocalcemia

1. Iron deficiency anemia, vitamin B12 and calcium deficiency

An advantage of using the NuvaRing vaginal ring for contraception is: 1. It does not require fitting and is easy to insert. 2. It is inserted once a week, eliminating the need to remember to take a daily pill. 3. Patients get a level of estrogen and progestin equal to combined oral contraceptives. 4. It also provides protection against vaginal infections.

1. It does not require fitting and is easy to insert.

Why is the consistency of taking paroxetine (Paxil) and never running out of medication more important than with most other selective serotonin reuptake inhibitors (SSRIs)? 1. It has a shorter half-life and withdrawal syndrome has a faster onset without taper. 2. It has the longest half-life and the withdrawal syndrome has a faster onset. 3. It is quasi-addictive in the dopaminergic reward system. 4. It is the most activating of SSRI medications and will cause the person to have sudden deep sadness.

1. It has a shorter half-life and withdrawal syndrome has a faster onset without taper.

What is considered the order of statin strength from lowest effect to highest? 1. Lovastatin, Simvastatin, Rosuvastatin 2. Rosuvastatin, Lovastatin, Atorvastatin 3. Atorvastatin, Rosuvastatin, Simvastatin 4. Simvastatin, Atorvastatin, Lovastatin

1. Lovastatin, Simvastatin, Rosuvastatin

Sadie is a 72-year-old patient who takes omeprazole for her chronic GERD. Chronic long-term omeprazole use places her at increased risk for: 1. Megaloblastic anemia 2. Osteoporosis 3. Hypertension 4. Strokes

1. Megaloblastic anemia

Sophie presents to the clinic with a malodorous vaginal discharge and is confirmed to have Trichomonas infection. Treatment for her would include: 1. Metronidazole 2 grams PO x 1 dose 2. Topical intravaginal metronidazole daily x 7 days 3. Intravaginal clindamycin daily x 7 days 4. Azithromycin 2 grams PO x 1 dose

1. Metronidazole 2 grams PO x 1 dose

Helima presents with a complaint of vaginal discharge that when tested meets the criteria for bacterial vaginosis. Treatment of bacterial vaginosis in nonpregnant symptomatic women would be: 1. Metronidazole 500 mg PO bid x 7 days 2. Doxycycline 100 mg PO bid x 7 days 3. Intravaginal tinidazole daily x 5 days 4. Metronidazole 2 grams PO x 1 dose

1. Metronidazole 500 mg PO bid x 7 days

One of the three types of heart failure involves systolic dysfunction. Potential causes of this most common form of heart failure include: 1. Myocardial ischemia and injury secondary to myocardial infarction 2. Inadequate relaxation and loss of muscle fiber secondary to valvular dysfunction 3. Increased demands of the heart beyond its ability to adapt secondary to anemia 4. Slower filling rate and elevated systolic pressures secondary to uncontrolled hypertension

1. Myocardial ischemia and injury secondary to myocardial infarction

Joanna had a small ventricle septal defect (VSD) repaired when she was 3 years old and has no residual cardiac problems. She is now 28 and is requesting prophylactic antibiotics for an upcoming dental visit. The appropriate antibiotic to prescribe according to current American College of Cardiology and American Heart Association guidelines is: 1. None, no antibiotic is required for dental procedures 2. Amoxicillin 2 grams 1 hour before the procedure 3. Ampicillin 2 grams IM or IV 30 minutes before the procedure 4. Azithromycin 1 gram 1 hour before the procedure

1. None, no antibiotic is required for dental procedures

John has clonidine, a centrally acting adrenergic blocker, prescribed for his hypertension. He should: 1. Not miss a dose or stop taking the drug because of potential rebound hypertension 2. Increase fiber in the diet to treat any diarrhea that may occur 3. Reduce fluid intake to less than 2 liters per day to prevent fluid retention 4. Avoid sitting for long periods, as this can lead to deep vein thrombosis

1. Not miss a dose or stop taking the drug because of potential rebound hypertension

Cost-effectiveness analysis compares two or more treatments or programs that are: 1. Not necessarily therapeutically equivalent 2. Considered equal in efficacy 3. Compared with the dollar value of the benefit received 4. Expressed in terms of patient preference or quality-adjusted life years

1. Not necessarily therapeutically equivalent

Factors common in women that can affect adherence to a treatment regimen include all of the following EXCEPT: 1. Number of drugs taken: Women tend to take fewer drugs over longer periods of time 2. Fear that medications can cause disease: Information obtained from social networks may be inaccurate for a specific woman 3. Nutritional status: Worries about possible weight gain from a given drug may result in nonadherence 4. Religious differences: A patient's belief system that is not congruent with the treatment regimen presents high risk for nonadherence

1. Number of drugs taken: Women tend to take fewer drugs over longer periods of time

Pharmacologic interventions are costly. Patients for whom the cost/benefit variable is especially important include: 1. Older adults and those on fixed incomes 2. Patients with chronic illnesses 3. Patients with copayments for drugs on their insurance 4. Patients on public assistance

1. Older adults and those on fixed incomes

Which of the following create a higher risk for digoxin toxicity? Both the cause and the reason for it must be correct. 1. Older adults because of reduced renal function 2. Administration of aldosterone antagonist diuretics because of decreased potassium levels 3. Taking an antacid for gastroesophageal reflux disease because it increases the absorption of digoxin 4. Doses between 0.25 and 0.5 mg/day

1. Older adults because of reduced renal function

An example of a first-dose reaction that may occur includes: 1. Orthostatic hypotension that does not occur with repeated doses 2. Purple glove syndrome with phenytoin use 3. Hemolytic anemia from ceftriaxone use 4. Contact dermatitis from neomycin use

1. Orthostatic hypotension that does not occur with repeated doses

Many patients self-medicate with antacids. Which patients should be counseled to not take calcium carbonate antacids without discussing it with their provider or a pharmacist first? 1. Patients with kidney stones 2. Pregnant patients 3. Patients with heartburn 4. Postmenopausal women

1. Patients with kidney stones

The U.S. Food and Drug Administration warns that androgens may cause: 1. Peliosis hepatis 2. Orthostatic hypotension 3. Menstrual irregularities 4. Acne

1. Peliosis hepatis

Zoe presents with genital warts present on her labia. Patient-applied topical therapy for warts includes: 1. Podofilox 0.5% gel 2. Podophyllin 10% resin 3. Trichloracetic acid 4. Any of the above

1. Podofilox 0.5% gel

Because of its action on various body systems, the patient taking a thiazide or loop diuretic may also need to receive the following supplement: 1. Potassium 2. Calcium 3. Magnesium 4. Phosphates

1. Potassium

It is reasonable to add ___________ to a Helicobacter pylori treatment regimen to improve eradication rates of H. pylori. 1. Probiotics 2. Omega-3 fatty acids 3. Plant sterols 4. Fiber

1. Probiotics

If a patient with H. pylori-positive peptic ulcer disease fails first-line therapy, the second-line treatment is: 1. Proton pump inhibitor bid plus metronidazole plus tetracycline plus bismuth subsalicylate for 14 days 2. Test H. pylori for resistance to common treatment regimens 3. Proton pump inhibitor plus clarithromycin plus amoxicillin for 14 days 4. Proton pump inhibitor and levofloxacin for 14 days

1. Proton pump inhibitor bid plus metronidazole plus tetracycline plus bismuth subsalicylate for 14 days

Ralph's blood pressure remains elevated despite increased doses of his drug. The NP is concerned that he might not be adhering to his treatment regimen. Which of the following events would suggest that he might not be adherent? 1. Ralph states that he always takes the drug "when I feel my pressure is going up." 2. Ralph contacts his NP to discuss the need to increase the dosage. 3. Ralph consistently keeps his follow-up appointments to check his blood pressure. 4. All of the above show that he is adherent to the drug regimen.

1. Ralph states that he always takes the drug "when I feel my pressure is going up."

Azithromycin dosing requires that the first day's dosage be twice those of the other 4 days of the prescription. This is considered a loading dose. A loading dose: 1. Rapidly achieves drug levels in the therapeutic range 2. Requires four or five half lives to attain 3. Is influenced by renal function 4. Is directly related to the drug circulating to the target tissues

1. Rapidly achieves drug levels in the therapeutic range

Shana received her first medroxyprogesterone (Depo Provera) injection 6 weeks ago and calls the clinic with a concern that she has been having a light "period" off and on since receiving her Depo shot. What would be the management of Shana? 1. Reassurance that some spotting is normal the first few months of Depo and it should improve. 2. Schedule an appointment for an exam as this is not normal. 3. Prescribe 4 weeks of estrogen to treat the abnormal vaginal bleeding. 4. Order a pregnancy test and suggest she use a back-up method of contraception until she has her next shot.

1. Reassurance that some spotting is normal the first few months of Depo and it should improve.

Beta blockers treat hypertension because they: 1. Reduce peripheral resistance 2. Vasoconstrict coronary arteries 3. Reduce norepinephrine 4. Reduce angiotensin II production

1. Reduce peripheral resistance

Many patients with hyperlipidemia are treated with more than one drug. Combining a fibric acid derivative such as gemfibrozil with which of the following is not recommended? The drug and the reason must both be correct for the answer to be correct. 1. Reductase inhibitors, due to an increased risk for rhabdomyolysis 2. Bile-acid sequestering resins, due to interference with folic acid absorption 3. Grapefruit juice, due to interference with metabolism 4. Niacin, due to decreased gemfibrozil activity

1. Reductase inhibitors, due to an increased risk for rhabdomyolysis

The U.S. Drug Enforcement Administration: 1. Registers manufacturers and prescribers of controlled substances 2. Regulates NP prescribing at the state level 3. Sanctions providers who prescribe drugs off-label 4. Provides prescribers with a number they can use for insurance billing

1. Registers manufacturers and prescribers of controlled substances

Milk and other foods that alkalinize the urine may: 1. Result in basic drugs being reabsorbed in the renal tubule 2. Increase the elimination of basic drugs in the urine 3. Decrease the elimination of acidic drugs 4. Not alter drug elimination due to the minimal change in urine pH

1. Result in basic drugs being reabsorbed in the renal tubule

What is a common side effect concern with hypertensive medications and all individuals, but especially the elderly? 1. Risk of falls 2. Triggering of a hypertensive crisis 3. Erectile priapism 4. Risk for bladder cancer development

1. Risk of falls

Suzanne is started on paroxetine (Paxil), a selective serotonin reuptake inhibitor (SSRI), for depression. Education regarding her antidepressant includes: 1. SSRIs may take 2 to 6 weeks before she will have maximum drug effects. 2. Red-green color blindness may occur and should be reported. 3. If she experiences dry mouth or heart rates greater than 80, she should stop taking the drug immediately. 4. She should eat lots of food high in fiber to prevent constipation.

1. SSRIs may take 2 to 6 weeks before she will have maximum drug effects.

Because of their site of action, bile acid sequestering resins: 1. Should be administered separately from other drugs by at least 4 hours 2. May increase the risk for bleeding 3. Both 1 and 2 4. Neither 1 nor 2

1. Should be administered separately from other drugs by at least 4 hours

Alterations in drug metabolism among Asians may lead to: 1. Slower metabolism of antidepressants, requiring lower doses 2. Faster metabolism of neuroleptics, requiring higher doses 3. Altered metabolism of omeprazole, requiring higher doses 4. Slower metabolism of alcohol, requiring higher doses

1. Slower metabolism of antidepressants, requiring lower doses

Which of the following substances is the most likely to be absorbed in the intestines rather than in the stomach? 1. Sodium bicarbonate 2. Ascorbic acid 3. Salicylic acid 4. Glucose

1. Sodium bicarbonate

Which the following persons should not have a statin medication ordered? 1. Someone with 3 first- or second-degree family members with history of muscle issues when started on statins 2. Someone with high lipids, but low BMI 3. Premenopausal woman with recent history of hysterectomy 4. Prediabetic male with known metabolic syndrome

1. Someone with 3 first- or second-degree family members with history of muscle issues when started on statins

Patients taking warfarin need to be educated about the vitamin K content of foods to avoid therapeutic failure. Foods high in vitamin K that should be limited to no more than one serving per day include: 1. Spinach 2. Milk 3. Romaine lettuce 4. Cauliflower

1. Spinach

Vera, age 70, has isolated systolic hypertension. Calcium channel blocker dosages for her should be: 1. Started at about half the usual dosage 2. Not increased over the usual dosage for an adult 3. Given once daily because of memory issues in the older adult 4. Withheld if she experiences gastroesophageal reflux

1. Started at about half the usual dosage

The topical patch combined contraceptive (Ortho Evra) is: 1. Started on the first day of the menstrual cycle 2. Recommended for women over 200 pounds 3. Not as effective as oral combined contraceptives 4. Known to have more adverse effects, such as nausea, than the oral combined contraceptives

1. Started on the first day of the menstrual cycle

Nicholas has been diagnosed with type A influenza. Appropriate prescribing of oseltamivir (Tamiflu) would include: 1. Starting oseltamivir within the first 48 hours of influenza symptoms 2. Advising the patient he can stop the oseltamivir when his symptoms resolve 3. Educating the patient that oseltamivir will cure influenza 4. Prophylactic treatment of all family members

1. Starting oseltamivir within the first 48 hours of influenza symptoms

Behaviors predictive of addiction to controlled substances include: 1. Stealing or borrowing another patient's drugs 2. Requiring increasing doses of opiates for pain associated with malignancy 3. Receiving refills of a Schedule II prescription on a regular basis 4. Requesting that only their own primary care provider prescribe for them

1. Stealing or borrowing another patient's drugs

A potentially life-threatening adverse response to angiotensin-converting enzyme inhibitors is angioedema. Which of the following statements is true about this adverse response? 1. Swelling of the tongue or hoarseness are the most common symptoms. 2. It appears to be related to the decrease in aldosterone production. 3. Presence of a dry, hacky cough indicates a high risk for this adverse response. 4. Because it takes time to build up a blood level, it occurs after being on the drug for about 1 week.

1. Swelling of the tongue or hoarseness are the most common symptoms.

Jamie was treated for chancroid. Follow-up testing after treatment of chancroid would be: 1. Syphilis and HIV testing at 3-month intervals 2. Chancroid-specific antigen test every 3 months 3. Urine testing for Haemophilus ducreyi in 3 to 6 months for test of cure 4. Annual HIV testing if engaging in high-risk sexual behavior

1. Syphilis and HIV testing at 3-month intervals

The timing of NSAIDS for best control of severe menstrual cramps includes: 1. Taking them for 2-3 days prior to the start of bleeding 2. Taking them 2-3 times a day during the first 2 days 3. Taking them every 2-3 hours 4. They have not been found to be helpful at all

1. Taking them for 2-3 days prior to the start of bleeding

Transdermal nicotine replacement (the patch) is an effective choice in tobacco cessation because: 1. The patch provides a steady level of nicotine without reinforcing oral aspects of smoking. 2. There is the ability to "fine tune" the amount of nicotine that is delivered to the patient at any one time. 3. There is less of a problem with nicotine toxicity than other forms of nicotine replacement. 4. Transdermal nicotine is safer in pregnancy.

1. The patch provides a steady level of nicotine without reinforcing oral aspects of smoking.

The longer-term Xanax patient comes in and states they need a higher dose of the medication. They deny any additional, new, or accelerating triggers of their anxiety. What is the probable reason? 1. They have become tolerant of the medication, which is characterized by the need for higher and higher doses. 2. They are a drug seeker. 3. They are suicidal. 4. They only need additional counseling on lifestyle modification.

1. They have become tolerant of the medication, which is characterized by the need for higher and higher doses.

Cara is taking levetiracetam (Keppra) to treat seizures. Routine education for levetiracetam includes reminding her: 1. To not abruptly discontinue levetiracetam due to risk for withdrawal seizures 2. To wear sunscreen due to photosensitivity from levetiracetam 3. To get an annual eye exam while on levetiracetam 4. To report weight loss if it occurs

1. To not abruptly discontinue levetiracetam due to risk for withdrawal seizures

Every antibiotic drug class has resistant organisms that influence prescribing decisions. 1. True 2. False

1. True

Anaphylactic shock is a: 1. Type I reaction, called immediate hypersensitivity reaction 2. Type II reaction, called cytotoxic hypersensitivity reaction 3. Type III allergic reaction, called immune complex hypersensitivity 4. Type IV allergic reaction, called delayed hypersensitivity reaction

1. Type I reaction, called immediate hypersensitivity reaction

Medication agreements or "Pain Medication Contracts" are recommended to be used: 1. Universally for all prescribing for chronic pain 2. For patients who have repeated requests for pain medication 3. When you suspect a patient is exhibiting drug-seeking behavior 4. For patients with pain associated with malignancy

1. Universally for all prescribing for chronic pain

Patricia has been prescribed doxycycline for a chlamydia infection. She is healthy and her only medication is an oral combined contraceptive. Patricia's education would include: 1. Use a back-up method of birth control (condom) until her next menses. 2. Doxycycline may cause tendonitis and she should report any joint pain. 3. Her partner will need treatment if her infection doesn't clear with the doxycycline. 4. Doxycycline is used for one-dose treatment of STIs; take the whole prescription at once.

1. Use a back-up method of birth control (condom) until her next menses.

To improve actual effectiveness of oral contraceptives women should be educated regarding: 1. Use of a back-up method if they have vomiting or diarrhea during a pill packet 2. Doubling pills if they have diarrhea during the middle of a pill pack 3. The fact that they will have a normal menstrual cycle if they miss two pills 4. The fact that mid-cycle spotting is not normal and the provider should be contacted immediately

1. Use of a back-up method if they have vomiting or diarrhea during a pill packet

Maternal-to-child transmission of HIV infection during pregnancy may be prevented by: 1. Use of antiviral drugs such as zidovudine 2. Use of condoms during intercourse 3. Both 1 and 2 4. Neither 1 nor 2

1. Use of antiviral drugs such as zidovudine

Combined alpha-beta antagonists are used to reduce the progression of heart failure because they: 1. Vasodilate the peripheral vasculature 2. Decrease cardiac output 3. Increase renal vascular resistance 4. Reduce atherosclerosis secondary to elevated serum lipoproteins

1. Vasodilate the peripheral vasculature

Nicotine has a variety of effects on nicotinic receptors throughout the body. Which of the following is NOT an effect of nicotine? 1. Vasodilation and decreased heart rate 2. Increased secretion of gastric acid and motility of the GI smooth muscle 3. Release of dopamine at the pleasure center 4. Stimulation of the locus coeruleus

1. Vasodilation and decreased heart rate

An appropriate first-line drug for the treatment of depression with fatigue and low energy would be: 1. Venlafaxine (Effexor) 2. Escitalopram (Lexapro) 3. Buspirone (Buspar) 4. Amitriptyline (Elavil)

1. Venlafaxine (Effexor)

What assessment that can be done at home is the most reliable for making decisions to change HF medications? 1. Weight 2. BP 3. Heart rate 4. Serum Glucose

1. Weight

Patients with psychiatric illnesses have adherence rates to their drug regimen between 35% and 60%. To improve adherence in this population, prescribe drugs: 1. With a longer half-life so that missed doses produce a longer taper on the drug curve 2. In oral formulations that are more easily taken 3. That do not require frequent monitoring 4. Combined with patient education about the need to adhere even when symptoms are absent

1. With a longer half-life so that missed doses produce a longer taper on the drug curve

Patients should be instructed regarding the rapid onset of zolpidem (Ambien) because: 1. Zolpidem should be taken just before going to bed. 2. Zolpidem may cause dry mouth and constipation. 3. Patients may need to double the dose for effectiveness. 4. They should stop drinking alcohol at least 30 minutes before taking zolpidem.

1. Zolpidem should be taken just before going to bed.

A patient may develop neutropenia from using topical Silvadene for burns. Neutropenia is a(n): 1. Cytotoxic hypersensitivity reaction 2. Immune complex hypersensitivity 3. Immediate hypersensitivity reaction 4. Delayed hypersensitivity reaction

1.Cytotoxic hypersensitivity reaction

The U.S. Food and Drug Administration approval is required for: 1. Medical devices, including artificial joints 2. Over-the-counter vitamins 3. Herbal products, such as St John's wort 4. Dietary supplements, such as Ensure

1.Medical devices, including artificial joints

The route of excretion of a volatile drug will likely be the: 1. Kidneys 2. Lungs 3. Bile and feces 4. Skin

2 lungs

Pharmacokinetics among Asians are universal to all the Asian ethnic groups. 1. True 2. False

2) False

Caffeine, exercise, and smoking should be avoided for at least how many minutes before blood pressure measurement? 1. 15 2. 30 3. 60 4. 90

2. 30

An NP would prescribe the liquid form of ibuprofen for a 6-year-old child because: 1. Drugs given in liquid form are less irritating to the stomach. 2. A 6-year-old child may have problems swallowing a pill. 3. Liquid forms of medication eliminate the concern for first-pass effect. 4. Liquid ibuprofen does not have to be dosed as often as the tablet form.

2. A 6-year-old child may have problems swallowing a pill.

Which of the following variables is a factor in drug absorption? 1. The smaller the surface area for absorption, the more rapidly the drug is absorbed. 2. A rich blood supply to the area of absorption leads to better absorption. 3. The less soluble the drug, the more easily it is absorbed. 4. Ionized drugs are easily absorbed across the cell membrane.

2. A rich blood supply to the area of absorption leads to better absorption.

The Vaccine Adverse Events Reporting System is: 1. A mandatory reporting system for all health-care providers when they encounter an adverse vaccine event 2. A voluntary reporting system that health-care providers or consumers may use to report vaccine adverse events 3. Utilized to send out safety alerts regarding emerging vaccine safety issues 4. Activated when a vaccine has been proven to cause significant adverse effects

2. A voluntary reporting system that health-care providers or consumers may use to report vaccine adverse events

Kasey fractured his ankle in two places and is asking for medication for his pain. The appropriate first-line medication would be: 1. Ibuprofen (Advil) 2. Acetaminophen with hydrocodone (Vicodin) 3. Oxycodone (Oxycontin) 4. Oral morphine (Roxanol)

2. Acetaminophen with hydrocodone (Vicodin)

Tetracyclines such as minocycline are safe to use in: 1. Pregnant women 2. Adolescents 3. Patients with renal dysfunction 4. Patients with hepatic dysfunction

2. Adolescents

Aldactone family medications are frequently used when the hypertensive patient also has: 1. Hyperkalemia 2. Advancing liver dysfunction 3. The need for birth control 4. Rheumatoid arthritis

2. Advancing liver dysfunction

All patients with hypertension benefit from diuretic therapy, but those who benefit the most are: 1. Those with orthostatic hypertension 2. African Americans 3. Those with stable angina 4. Diabetics

2. African Americans

Therapeutic drug levels are drawn when a drug reaches steady state. Drugs reach steady state: 1. After the second dose 2. After four to five half-lives 3. When the patient feels the full effect of the drug 4. One hour after IV administration

2. After four to five half-lives

Besides prescribing antimicrobial therapy, patients with bacterial vaginosis require education regarding the fact that: 1. The most recent partners in the past 60 days should also be treated. 2. Alcohol should not be consumed during and for 1 day after metronidazole is taken. 3. Condoms should be used during intercourse if intravaginal clindamycin cream is used. 4. Co-treatment for chlamydia is necessary.

2. Alcohol should not be consumed during and for 1 day after metronidazole is taken.

Sarah is a 25-year-old female who is 8 weeks pregnant and has a urinary tract infection. What would be the appropriate antibiotic to prescribe for her? 1. Ciprofloxacin (Cipro) 2. Amoxicillin (Trimox) 3. Doxycycline 4. Trimethoprim-sulfamethoxazole (Septra)

2. Amoxicillin (Trimox)

Digoxin has a very limited role in treatment of heart failure. It is used mainly for patients with: 1. Ejection fractions above 40% 2. An audible S3 3. Mitral stenosis as a primary cause for heart failure 4. Renal insufficiency

2. An audible S3

Drugs that are receptor antagonists, such as beta blockers, may cause: 1. Down regulation of the drug receptor 2. And exaggerated response if abruptly discontinued 3. Partial blockade of the effects of agonist drugs 4. An exaggerated response to competitive drug agonsits

2. An exaggerated response if abruptly discontinued

Sarah, a 42-year-old female, requests a prescription for an anorexiant to treat her obesity. A trial of phentermine is prescribed. Prescribing precautions include: 1. Understanding that obesity is a contraindication to prescribing phentermine 2. Anorexiants may cause tolerance and should only be prescribed for 6 months 3. Patients should be monitored for postural hypotension 4. Renal function should be monitored closely while on anorexiants

2. Anorexiants may cause tolerance and should only be prescribed for 6 months

There is evidence that dietary supplementation or adequate intake of fish oils and omega-3 fatty acids have well-documented: 1. Concern for developing cardiac dysrhythmias 2. Anti-inflammatory effects 3. Total cholesterol-lowering effects 4. Effects on fasting blood sugar

2. Anti-inflammatory effects

Common over-the-counter pain relievers such as acetaminophen or ibuprofen: 1. Are always safer for the patient than prescription pain medication 2. Are harmful if taken in higher than recommended amounts 3. Have minimal interaction with prescription medications 4. Should never be given to children unless recommended by their provider

2. Are harmful if taken in higher than recommended amounts

Sook has been prescribed gabapentin to treat neuropathic pain and is complaining of feeling depressed and having "strange" thoughts. The appropriate initial action would be: 1. Increase her dose 2. Assess for suicidal ideation 3. Discontinue the medication immediately 4. Decrease her dose to half then slowly titrate up the dose

2. Assess for suicidal ideation

Janet was recently treated with clindamycin for an infection. She calls the advice nurse because she is having frequent diarrhea that she thinks may have blood in it. What would be the appropriate care for her? 1. Encourage increased fluids and fiber. 2. Assess her for pseudomembranous colitis. 3. Advise her to eat yogurt daily to help restore her gut bacteria. 4. Start her on an antidiarrheal medication.

2. Assess her for pseudomembranous colitis.

The most common cause of angina is: 1. Vasospasm of the coronary arteries 2. Atherosclerosis 3. Platelet aggregation 4. Low systemic oxygen

2. Atherosclerosis

There is sufficient evidence to support the use of omega-3 fatty acids to treat the following disease(s): 1. Asthma 2. Autism 3. Arthritis 4. All of the above

2. Autism

Treatment goals in prescribing should: 1. Always be curative 2. Be patient-centered 3. Be convenient for the provider 4. Focus on the cost of therapy

2. Be patient-centered

Serum digoxin levels are monitored for potential toxicity. Monitoring should occur: 1. Within 6 hours of the last dose 2. Because a reference point is needed in adjusting a dose 3. After three half-lives from the starting of the drug 4. When a patient has stable renal function

2. Because a reference point is needed in adjusting a dose

Drugs administered via IV; 1. Need to be lipid soluble in order to be easily absorbed 2. Begin distribution into the body immediately 3. Are easily absorbed if they are non ionized 4. May use pinocytosis to be absorbed

2. Begin distribution into the body immediiately

The drug of choice for treatment of early latent or tertiary syphilis is: 1. Ceftriaxone IM 2. Benzathine penicillin G IM 3. Oral azithromycin 4. Oral ciprofloxacin

2. Benzathine penicillin G IM

The drug of choice for treatment of primary or secondary syphilis is: 1. Ceftriaxone IM 2. Benzathine penicillin G IM 3. Oral azithromycin 4. Oral ciprofloxacin

2. Benzathine penicillin G IM

The NP chooses to give cephalexin every 8 hours based on knowledge of the drug's: 1. Propensity to go to the target receptor 2. Biological half-life 3. Pharmacodynamics 4. Safety and side effects

2. Biological half-life

Patrick is a 10-year-old patient who presents with uncomfortable constipation. Along with diet changes, a laxative is ordered to provide more rapid relief of constipation. An appropriate choice of medication for a 10-year-old child would be: 1. PEG 3350 (Miralax) 2. Bisacodyl (Dulcolax) suppository 3. Docusate (Colace) suppository 4. Polyethylene glycol electrolyte solution

2. Bisacodyl (Dulcolax) suppositor

Hannah will be traveling to Mexico with her church group over spring break to build houses. She is concerned she may develop traveler's diarrhea. Advice includes following normal food and water precautions as well as taking: 1. Loperamide four times a day throughout the trip 2. Bismuth subsalicylate with each meal and at bedtime 3. A prescription for diphenoxylate with atropine to use if she gets diarrhea 4. None of the above

2. Bismuth subsalicylate with each meal and at bedtime

When comparing angiotensin-converting enzyme (ACE) and angiotensin II receptor blocker (ARB) medications, which of the following holds true? 1. Both have major issues with a dry, irritating cough 2. Both contribute to some retention of potassium 3. ARBs have a stronger impact on hypertension control than ACE medications 4. ARBs have stronger diabetes mellitus renal protection properties than ACE medications

2. Both contribute to some retention of potassium

Patients taking hormonal contraceptives and hormone replacement therapy need to take the drug daily at the same time to prevent: 1. Nausea 2. Breakthrough bleeding 3. Breast tenderness 4. Pregnancy

2. Breakthrough bleeding

Lifestyle changes are the first step in treatment of gastroesophageal reflux disease (GERD). Food or drink that may aggravate GERD include: 1. Eggs 2. Caffeine 3. Chocolate 4. Soda pop

2. Caffeine

Dwayne has recently started on carbamazepine to treat seizures. He comes to see you and you note that while his carbamazepine levels had been in the therapeutic range, they are now low. The possible cause for the low carbamazepine levels include: 1. Dwayne hasn't been taking his carbamazepine because it causes insomnia. 2. Carbamazepine auto-induces metabolism, leading to lower levels in spite of good compliance. 3. Dwayne was not originally prescribed the correct amount of carbamazepine. 4. Carbamazepine is probably not the right antiseizure medication for Dwayne.

2. Carbamazepine auto-induces metabolism, leading to lower levels in spite of good compliance.

Immunomodulators such as azathioprine may cause a delayed adverse drug reaction known as a type D reaction because they are known: 1. Teratogens 2. Carcinogens 3. To cause hypersensitivity reactions 4. Hypothalamus-pituitary-adrenal axis suppressants

2. Carcinogens

Under new U.S. Food and Drug Administration labeling, Pregnancy Categories will be: 1. Strengthened with a new coding such as C+ or C- to discern when a drug is more or less toxic to the fetus 2. Changed to incorporate a pregnancy risk summary and clinical considerations on the drug label 3. Eliminated, and replaced with a link to the National Library of Medicine TOXNET Web site for in-depth information regarding pregnancy concerns 4. Clarified to include information such as safe dosages in each trimester of pregnancy

2. Changed to incorporate a pregnancy risk summary and clinical considerations on the drug label

Education when prescribing androgens to male patients includes advising that: 1. Short-term use places the patient at risk for hepatocellular carcinoma. 2. Cholestatic hepatitis and jaundice may occur with low doses of androgens. 3. Gynecomastia is a rare occurrence with the use of androgens. 4. Low sperm levels only occur with long-term use of androgens.

2. Cholestatic hepatitis and jaundice may occur with low doses of androgens.

Absolute contraindications to estrogen therapy include: 1. History of any type of cancer 2. Clotting disorders 3. History of tension headaches 4. Orthostatic hypotension

2. Clotting disorders

A laboratory result indicates that the peak level for a drug is above the minimum toxic concentration. This means that the: 1. Concentration will produce therapeutic effects 2. Concentration will produce an adverse response 3. Time between doses must be shortened 4. Duration of action of the drug is too long

2. Concentration will produce an adverse response

Criteria for choosing an effective drug for a disorder include: 1. Asking the patient what drug they think would work best for them. 2. Consulting nationally recognized guidelines for disease management 3. Prescribing medications that are available as samples before writing a prescription 4. Following U.S. DEA guidelines for prescribing.

2. Consulting nationally recognized guidelines for disease management

When prescribing metronidazole (Flagyl) to treat bacterial vaginosis, patient education would include: 1. Metronidazole is safe in the first trimester of pregnancy. 2. Consuming alcohol in any form may cause a severe reaction. 3. Sexual partners need concurrent therapy. 4. Headaches are a sign of a serious adverse reaction and need immediate evaluation.

2. Consuming alcohol in any form may cause a severe reaction.

The ailment that generates the greatest over-the-counter annual drug sales is: 1. Constipation 2. Cough and colds 3. Heartburn 4. Acute and chronic pain

2. Cough and colds

Niaspan is less likely to cause which side effect that is common to niacin? 1. Gastrointestinal irritation 2. Cutaneous flushing 3. Dehydration 4. Headaches

2. Cutaneous flushing

Beta blockers are the drugs of choice for exertional angina because they: 1. Improve myocardial oxygen supply by vasodilating the coronary arteries 2. Decrease myocardial oxygen demand by decreasing heart rate and vascular resistance 3. Both 1 and 2 4. Neither 1 nor 2

2. Decrease myocardial oxygen demand by decreasing heart rate and vascular resistance

When prescribing medroxyprogesterone (Depo Provera) injections, essential education would include advising of the following potential adverse drug effects: 1. Hypertension and dysuria 2. Depression and weight gain 3. Abdominal pain and constipation 4. Orthostatic hypotension and dermatitis

2. Depression and weight gain

An appropriate drug to initially treat panic disorder is: 1. Alprazolam (Xanax) 2. Diazepam (Valium) 3. Buspirone (Buspar) 4. Amitriptyline (Elavil)

2. Diazepam (Valium)

You are managing the care of a patient recently diagnosed with benign prostatic hyperplasia (BPH). He is taking tamsulosin but reports dizziness when standing abruptly. The best option for this patient is: 1. Continue the tamsulosin because the side effect will resolve with continued treatment. 2. Discontinue the tamsulosin and start doxazosin. 3. Have him double his fluid intake and stand more slowly. 4. Prescribe meclizine as needed for the dizziness.

2. Discontinue the tamsulosin and start doxazosin.

Patient education regarding prescribed medication includes: 1. Instructions written at the high school reading level 2. Discussion of expected adverse drug reactions 3. How to store leftover medication such as antibiotics 4. Verbal instructions always in English

2. Discussion of expected adverse drug reactions

Despite good blood pressure control, an NP might change a patient's drug from an angiotensin-converting enzyme (ACE) inhibitor to an angiotensin II receptor blocker (ARB) because the ARB: 1. Is stronger than the ACE inhibitor 2. Does not produce a dry, hacky cough 3. Has no effect on the renal system 4. Reduces sodium and water retention

2. Does not produce a dry, hacky cough

Adherence is improved when a drug can be given once daily. Which of the following drugs can be given once daily? 1. Tacrine 2. Donepezil 3. Memantine 4. Pyridostigmine

2. Donepezil

Tom is taking lithium for bipolar disorder. He should be taught to: 1. Take his lithium with food 2. Eat a diet with consistent levels of salt (sodium) 3. Drink at least 2 quarts of water if he is in a hot environment 4. Monitor blood glucose levels

2. Eat a diet with consistent levels of salt (sodium)

Since 40% of bone accrual occurs during adolescence, building bone during this time is critical. Ways to improve bone accrual in adolescents include: 1. Use of bisphosphonates early if dual energy X-ray absorptiometry (DEXA) scans show limited bone accrual 2. Encouraging a daily dietary intake of 1,300 mg of calcium and 400 IU of vitamin D 3. Avoiding all birth control methods that include progesterone 4. Fostering the intake of iron mainly in green and leafy vegetables

2. Encouraging a daily dietary intake of 1,300 mg of calcium and 400 IU of vitamin D

The metabolism of drugs in women is primarily impacted by: 1. Hepatic blow flow 2. Enzymes of the CYP450 system differences with men 3. The amount of gastric secretions 4. Whether they are pre- or postmenopausal

2. Enzymes of the CYP450 system differences with men

An appropriate drug for the treatment of depression with anxiety would be: 1. Alprazolam (Xanax) 2. Escitalopram (Lexapro) 3. Buspirone (Buspar) 4. Amitriptyline (Elavil)

2. Escitalopram (Lexapro)

Gastroesophageal reflux disease may be aggravated by the following medication that affects lower esophageal sphincter (LES) tone: 1. Calcium carbonate 2. Estrogen 3. Furosemide 4. Metoclopramide

2. Estrogen

Felicity has been prescribed colestipol to treat her hyperlipidemia. Unlike other anti-lipidemics, this drug: 1. Blocks synthesis of cholesterol in the liver 2. Exchanges chloride ions for negatively charged acids in the bowel 3. Increases HDL levels the most among the classes 4. Blocks the lipoprotein lipase pathway

2. Exchanges chloride ions for negatively charged acids in the bowel

Research has shown that when patients who are covered by Medicare Part D reach the "donut hole" in coverage they: 1. Ask for extra refills of medication to get them through the months of no coverage 2. Fill their prescriptions less frequently, including critical medications such as warfarin or a statin 3. Fill their critical medications, but hold off on filling less-critical medications 4. Demonstrate no change in their prescription filling pattern

2. Fill their prescriptions less frequently, including critical medications such as warfarin or a statin

If a patient is exhibiting signs of nicotine toxicity when using transdermal nicotine, they should remove the patch and: 1. Wash the area thoroughly with soap and water. 2. Flush the area with clear water. 3. Reapply a new patch in 8 hours. 4. Take acetaminophen for the headache associated with toxicity.

2. Flush the area with clear water.

When a medication is added to a regimen for a synergistic effect, the combined effect of the drug is: 1. The sum of the effects of each drug individually 2. Greater than the sum of the effects of each drug individually 3. Less that the effect of each drug individually 4. Not predictable, as it varies with each individual use

2. Greater than the sum of the effects of each drug individually

The time required for the amount of drug in the body to decrease by 50% is called: 1. Steady state 2. Half-life 3. Phase II metabolism 4. Reduced bioavailability time

2. Half-life

Women are now the fastest growing population with HIV infection and AIDS. HIV-infected women: 1. Are less likely to become pregnant or to carry a pregnancy to term 2. Have higher rates of cervical dysplasia and HPV-concurrent infections 3. Are most often over 35 years of age 4. Most often come from Asian and Caucasian ethnic groups

2. Have higher rates of cervical dysplasia and HPV-concurrent infections

The patient shares with the provider that he is taking his Prozac at night before going to bed. What is the best response? 1. This is a good idea because this class of medications generally makes people sleepy. 2. Have you noticed that you are having more sleep issues since you started that? 3. This a good way to remember to take your daily medications because it is near your toothbrush. 4. This is a good plan because you can eat grapefruit if there is 8-12 hours difference in the time each are ingested.

2. Have you noticed that you are having more sleep issues since you started that?

David is a 34-year-old patient who is starting on paroxetine (Paxil) for depression. David's education regarding his medication would include: 1. Paroxetine may cause intermittent diarrhea. 2. He may experience sexual dysfunction beginning a month after he starts therapy. 3. He may have constipation and he should increase fluids and fiber. 4. Paroxetine has a long half-life so he may occasionally skip a dose.

2. He may experience sexual dysfunction beginning a month after he starts therapy.

Scott's seizures are well controlled on topiramate and he wants to start playing baseball. Education for Scott regarding his topiramate includes: 1. He should not play sports due to the risk of increased seizures 2. He should monitor his temperature and ability to sweat in the heat while playing 3. Reminding him that he may need higher dosages of topiramate when exercising 4. Encouraging him to use sunscreen due to photosensitivity from topiramate

2. He should monitor his temperature and ability to sweat in the heat while playing

Art is a 55-year-old smoker who has been diagnosed with angina and placed on nitrates. He complains of headaches after using his nitrate. An appropriate reply might be: 1. This is a parasympathetic response to the vasodilating effects of the drug. 2. Headaches are common side effects with these drugs. How severe are they? 3. This is associated with your smoking. Let's work on having you stop smoking. 4. This is not related to your medication. Are you under a lot of stress?

2. Headaches are common side effects with these drugs. How severe are they?

Ethnic differences have been found in drug: 1. Absorption 2. Hepatic metabolism 3. Filtration at the glomerulus 4. Passive tubular reabsorption

2. Hepatic metabolism

Gender differences between men and women in pharmacokinetics include: 1. More rapid gastric emptying so that drugs absorbed in the stomach have less exposure to absorption sites 2. Higher proportion of body fat so that lipophilic drugs have relatively greater volumes of distribution 3. Increased levels of bile acids so that drugs metabolized in the intestine have higher concentrations 4. Slower organ blood flow rates so drugs tend to take longer to be excreted

2. Higher proportion of body fat so that lipophilic drugs have relatively greater volumes of distribution

The New York Heart Association and the Canadian Cardiovascular Society have described grading criteria for levels of angina. Angina that occurs with unusually strenuous activity or on walking or climbing stair after meals is class: 1. I 2. II 3. III 4. IV

2. II

Demione is a 24-year-old patient who is 32 weeks pregnant and has tested positive for syphilis. The best treatment for her would be: 1. IM ceftriaxone 2. IM benzathine penicillin G 3. Oral azithromycin 4. Any of the above

2. IM benzathine penicillin G

Which of the following adverse effects are less likely in a beta1-selective blocker? 1. Dysrhythmias 2. Impaired insulin release 3. Reflex orthostatic changes 4. Decreased triglycerides and cholesterol

2. Impaired insulin release

The intangible costs of drug therapy include: 1. Loss of wages while undergoing therapy 2. Inconvenience, pain, and suffering incurred with therapy 3. Cost of medical equipment in the laboratory used to monitor therapeutic drug levels 4. Cost of prescription drug coverage, such as Medicare Part D

2. Inconvenience, pain, and suffering incurred with therapy

Prescribing less-expensive generic drugs or drugs off the $4 retail pharmacy lists: 1. Increases the complexity of the pharmacoeconomics of prescribing for the individual patient 2. Increases compliance by reducing the financial burden of drug costs to the patient 3. Is not sound prescribing practice due to the inferiority of the generic products 4. Will increase the overall cost of drugs to the system due to the ease of overprescribing less-expensive drugs

2. Increases compliance by reducing the financial burden of drug costs to the patient

Antacids treat gastroesophageal reflux disease by: 1. Increasing lower esophageal tone 2. Increasing gastric pH 3. Inhibiting gastric acid secretion 4. Increasing serum calcium level

2. Increasing gastric pH

When determining drug treatment the NP prescriber should: 1. Always use evidence-based guidelines 2. Individualize the drug choice for the specific patient 3. Rely on his or her experience when prescribing for complex patients 4. Use the newest drug on the market for the condition being treated

2. Individualize the drug choice for the specific patient

Grapefruit juice contains furanocoumarins that have been found to: 1. Alter absorption of drugs through competition for binding sites 2. Inhibit CYP 3A4, leading to decreased first-pass metabolism of drugs 3. Alter vitamin K metabolism, leading to prolonged bleeding 4. Enhance absorption of calcium and vitamin D

2. Inhibit CYP 3A4, leading to decreased first-pass metabolism of drugs

Drug antagonism is best defined as an effect of a drug that: 1. Leads to major physiological and psychological dependence 2. Is modified by the concurrent administration of another drug 3. Cannot be metabolized before another dose is administered 4. Leads to a decreased physiological response when combined with another drug

2. Is modified by the concurrent administration of another drug

The racial difference in drug pharmacokinetics seen in American Indian or Alaskan Natives are: 1. Increased CYP 2D6 activity, leading to rapid metabolism of some drugs 2. Largely unknown due to lack of studies of this population 3. Rapid metabolism of alcohol, leading to increased tolerance 4. Decreased elimination of opioids, leading to increased risk for addiction

2. Largely unknown due to lack of studies of this population

The choice of diuretic to use in treating hypertension is based on: 1. Presence of diabetes with loop diuretics being used for these patients 2. Level of kidney function with a thiazide diuretic being used for an estimated glomerular filtration rate higher than the mid-40mL/min range 3. Ethnicity with aldosterone antagonists best for African Americans and older adults 4. Presence of hyperlipidemia with higher doses needed for patients with LDL above 130 mg/dL

2. Level of kidney function with a thiazide diuretic being used for an estimated glomerular filtration rate higher than the mid-40mL/min range

Monitoring for a patient who is using androgens includes evaluation of: 1. Complete blood count and C-reactive protein levels 2. Lipid levels and liver function tests 3. Serum potassium and magnesium levels 4. Urine protein and potassium levels

2. Lipid levels and liver function tests

Monitoring of a patient who is on a lipid-lowering drug includes: 1. Fasting total cholesterol every 6 months 2. Lipid profile with attention to serum LDL 6 to 8 weeks after starting therapy, then again in 6 weeks 3. Complete blood count, C-reactive protein, and erythrocyte sedimentation rate after 6 weeks of therapy 4. All of the above

2. Lipid profile with attention to serum LDL 6 to 8 weeks after starting therapy, then again in 6 weeks

Before starting therapy with a statin, the following baseline laboratory values should be evaluated: 1. Complete blood count 2. Liver function (ALT/AST) and creatine kinase 3. C-reactive protein 4. All of the above

2. Liver function (ALT/AST) and creatine kinase

Oral emergency contraception (Plan B) is contraindicated in women who: 1. Had intercourse within the past 72 hours 2. May be pregnant 3. Are taking combined oral contraceptives 4. Are using a diaphragm

2. May be pregnant

Drugs that are designated Schedule II by the U.S. Drug Enforcement Administration: 1. Are known teratogens during pregnancy 2. May not be refilled; a new prescription must be written 3. Have a low abuse potential 4. May be dispensed without a prescription unless regulated by the state

2. May not be refilled; a new prescription must be written

Which of the following drugs used to treat Alzheimer's disease is not an anticholinergic? 1. Donepezil 2. Memantine 3. Rivastigmine 4. Galantamine

2. Memantine

Anticholinesterase inhibitors are used to treat: 1. Peptic ulcer disease 2. Myasthenia gravis 3. Both 1 and 2 4. Neither 1 nor 2

2. Myasthenia gravis

When prescribing acyclovir, patients should be educated regarding the: 1. High risk of developing diarrhea 2. Need to drink lots of fluids during treatment 3. Risk for life-threatening rash such as Stevens-Johnson 4. Eccentric dosing schedule

2. Need to drink lots of fluids during treatment

Instructions for a patient who is starting nicotine replacement therapy include: 1. Smoke less than 10 cigarettes a day when starting nicotine replacement. 2. Nicotine replacement will help with the withdrawal cravings associated with quitting tobacco. 3. Nicotine replacement can be used indefinitely. 4. Nicotine replacement therapy is generally safe for all patients.

2. Nicotine replacement will help with the withdrawal cravings associated with quitting tobacco.

Isosorbide dinitrate is prescribed for a patient with chronic stable angina. This drug is administered twice daily, but the schedule is 7 a.m. and 2 p.m. because: 1. It is a long-acting drug with potential for toxicity. 2. Nitrate tolerance can develop. 3. Orthostatic hypotension is a common adverse effect. 4. It must be taken with milk or food.

2. Nitrate tolerance can develop.

The benefits to the patient of having an APRN prescriber include: 1. Nurses know more about Pharmacology than other prescribers because they take it both in their basic nursing program and in their APRN program 2. Nurses care for the patient from a holistic approach and include the patient in decision making regarding their care 3. APRNs are less likely to prescribe narcotics and other controlled substances 4. APRNs are able to prescribe independently in all states, whereas a PA needs to have a physician supervising their practice.

2. Nurse care for the patient from a holistic approach and include the patient in decision making regarding their care

Rodrigo has been prescribed procainamide after a myocardial infarction. He is monitored for dyspnea, jugular venous distention, and peripheral edema because they may indicate: 1. Widening of the area of infarction 2. Onset of congestive heart failure 3. An electrolyte imbalance involving potassium 4. Renal dysfunction

2. Onset of congestive heart failure

Pregnant patients who are taking isoniazid (INH) should take 25 mg/day of vitamin B6 (pyridoxine) to prevent: 1. Beriberi 2. Peripheral neuropathy 3. Rickets 4. Megaloblastic anemia

2. Peripheral neuropathy

Phase IV clinical trials in the United States are also known as: 1. Human bioavailability trials 2. Postmarketing research 3. Human safety and efficacy studies 4. The last stage of animal trials before the human trials begin

2. Postmarketing research

Anticholinergic agents, such as benztropine (Cogentin), may be given with a phenothiazine to: 1. Reduce the chance of tardive dyskinesia 2. Potentiate the effects of the drug 3. Reduce the tolerance that tends to occur 4. Increase central nervous system (CNS) depression

2. Potentiate the effects of the drug

Commercials on TV for erectile dysfunction (ED) medications warn about mixing them with nitrates. Why? 1. Increased risk of priapism 2. Profound hypotension 3. Development of blue discoloration to the visual field 4. Inactivation of the ED medication effect

2. Profound hypotension

The mechanism of action of oral combined contraceptives that prevents pregnancy is: 1. Estrogen prevents the luteinizing hormone surge necessary for ovulation. 2. Progestins thicken cervical mucus and slow tubal motility. 3. Estrogen thins the endometrium making implantation difficult. 4. Progestin suppresses follicle stimulating hormone release.

2. Progestins thicken cervical mucus and slow tubal motility.

If a patient with symptoms of gastroesophageal reflux disease states that he has been self-treating at home with OTC ranitidine daily, the appropriate treatment would be: 1. Prokinetic (metoclopramide) for 4 to 8 weeks 2. Proton pump inhibitor (omeprazole) for 12 weeks 3. Histamine2 receptor antagonist (ranitidine) for 4 to 8 weeks 4. Cytoprotective drug (misoprostol) for 2 weeks

2. Proton pump inhibitor (omeprazole) for 12 weeks

If a patient with gastroesophageal reflux disease who is taking a proton pump inhibitor daily is not improving, the plan of care would be: 1. Prokinetic (metoclopramide) for 8 to 12 weeks 2. Proton pump inhibitor (omeprazole) twice a day for 4 to 8 weeks 3. Histamine2 receptor antagonist (ranitidine) for 4 to 8 weeks 4. Cytoprotective drug (misoprostol) for 4 to 8 weeks

2. Proton pump inhibitor (omeprazole) twice a day for 4 to 8 weeks

Which of the following drugs should be used only when clearly needed in pregnant and breastfeeding women? 1. Memantine 2. Pyridostigmine 3. Galantamine 4. Rivastigmine

2. Pyridostigmine

To prevent the development of peripheral neuropathy in patients taking isoniazid for tuberculosis the patient is also prescribed: 1. Niacin (vitamin B3) 2. Pyridoxine (vitamin B6) 3. Riboflavin (vitamin B2) 4. Thiamine (vitamin B1)

2. Pyridoxine (vitamin B6)

Jim presents with complaints of "heartburn" that is minimally relieved with Tums (calcium carbonate) and is diagnosed with gastroesophageal reflux disease (GERD). An appropriate first-step therapy would be: 1. Omeprazole (Prilosec) twice a day 2. Ranitidine (Zantac) twice a day 3. Famotidine (Pepcid) once a day 4. Metoclopramide (Reglan) four times a day

2. Ranitidine (Zantac) twice a day

Antonia is a 3-year-old child who has a history of status epilepticus. Along with her routine antiseizure medication, she should also have a home prescription for_________ to be used for an episode of status epilepticus. 1. IV phenobarbital 2. Rectal diazepam (Diastat) 3. IV phenytoin (Dilantin) 4. Oral carbamazepine (Tegretol)

2. Rectal diazepam (Diastat)

Jaycee has been on escitalopram (Lexapro) for a year and is willing to try tapering off of the selective serotonin reuptake inhibitor. What is the initial dosage adjustment when starting a taper off antidepressants? 1. Change dose to every other day dosing for a week 2. Reduce dose by 50% for 3 to 4 days 3. Reduce dose by 50% every other day 4. Escitalopram (Lexapro) can be stopped abruptly due to its long half-life

2. Reduce dose by 50% for 3 to 4 days

Vitamin B2 (riboflavin) may be prescribed to: 1. Decrease the incidence of beriberi 2. Reduce headaches and migraines 3. Prevent pernicious anemia 4. Treat hyperlipidemia

2. Reduce headaches and migraines

Management of all types and grades of angina includes the use of lifestyle modification to reduce risk factors. Which of these modifications are appropriate for which reason? Both the modification and the reason for it must be true for the answer to be correct. 1. Lose at least 10 pounds of body weight. Excessive weight increases cardiac workload. 2. Reduce sodium intake to no more than 2,400 mg of sodium. Sodium increases blood volume and cardiac workload. 3. Increase potassium intake to at least 100 mEq/d. The heart needs higher levels of potassium to improve contractility and oxygen supply. 4. Intake a moderate amount of alcohol. Moderate intake has been shown by research to improve cardiac function.

2. Reduce sodium intake to no more than 2,400 mg of sodium. Sodium increases blood volume and cardiac workload.

Jim is being treated for hypertension. Because he has a history of heart attack, the drug chosen is atenolol. Beta blockers treat hypertension by: 1. Increasing heart rate to improve cardiac output 2. Reducing vascular smooth muscle tone 3. Increasing aldosterone-mediated volume activity 4. Reducing aqueous humor production

2. Reducing vascular smooth muscle tone

The next step in treatment when a patient has been on proton pump inhibitors twice daily for 12 weeks and not improving is: 1. Add a prokinetic (metoclopramide) 2. Referral for endoscopy 3. Switch to another proton pump inhibitor 4. Add a cytoprotective drug

2. Referral for endoscopy

Compelling indications for an ACE inhibitor as treatment for hypertension based on clinical trials includes: 1. Pregnancy 2. Renal parenchymal disease 3. Stable angina 4. Dyslipidemia

2. Renal parenchymal disease

Prescribing for women during their childbearing years requires constant awareness of the possibility of: 1. Pregnancy unless the women is on birth control 2. Risk for silent bacterial or viral infections of the genitalia 3. High risk for developmental disorders in their infants 4. Decreased risk for abuse during this time

2. Risk for silent bacterial or viral infections of the genitalia

Zainab is taking lamotrigine (Lamictal) and presents to the clinic with fever and lymphadenopathy. Initial evaluation and treatment includes: 1. Reassuring her she has a viral infection and to call if she isn't better in 4 or 5 days 2. Ruling out a hypersensitivity reaction that may lead to multi-organ failure 3. Rapid strep test and symptomatic care if strep test is negative 4. Observation only, with further assessment if she worsens

2. Ruling out a hypersensitivity reaction that may lead to multi-organ failure

Furosemide is added to a treatment regimen for heart failure that includes digoxin. Monitoring for this combination includes: 1. Hemoglobin 2. Serum potassium 3. Blood urea nitrogen 4. Serum glucose

2. Serum potassium

Kelly has diarrhea and is wondering if she can take loperamide (Imodium) for the diarrhea. Loperamide: 1. Can be given to patients of all ages, including infants and children, for viral gastroenteritis 2. Slows gastric motility and reduces fluid and electrolyte loss from diarrhea 3. Is the treatment of choice for the diarrhea associated with E. coli 0157 4. May be used in pregnancy and by lactating women

2. Slows gastric motility and reduces fluid and electrolyte loss from diarrhea

If a patient wants to quit smoking, nicotine replacement therapy is recommended if the patient: 1. Smokes more than 10 cigarettes a day 2. Smokes within 30 minutes of awakening in the morning 3. Smokes when drinking alcohol 4. All of the above

2. Smokes within 30 minutes of awakening in the morning

Clinical dosing of Bethanechol: 1. Begins at the highest effective dose to obtain a rapid response 2. Starts at 5 mg to 10 mg PO and is repeated every hour until a satisfactory clinical response is achieved 3. Requires dosing only once daily 4. Is the same for both the oral and parenteral route

2. Starts at 5 mg to 10 mg PO and is repeated every hour until a satisfactory clinical response is achieved

Scopolamine can be used to prevent the nausea and vomiting associated with motion sickness. The patient is taught to: 1. Apply the transdermal disk at least 4 hours before the antiemetic effect is desired. 2. Swallow the tablet 1 hour before traveling where motion sickness is possible. 3. Place the tablet under the tongue and allow it to dissolve. 4. Change the transdermal disk daily for maximal effect.

2. Swallow the tablet 1 hour before traveling where motion sickness is possible.

According to the 2003-2006 National Health and Nutrition Examination Survey study of dietary intake, the group at highest risk for inadequate calcium intake was: 1. The elderly (over age 60 years) 2. Teenage females 3. Teenage males 4. Preschoolers

2. Teenage females

When prescribing temazepam (Restoril) for insomnia, patient education includes: 1. Take temazepam nightly approximately 15 minutes before bedtime. 2. Temazepam should not be used more than three times a week for less than 3 months. 3. Drinking 1 ounce of alcohol will cause additive effects and the patient will sleep better. 4. Exercise for at least 30 minutes within 2 hours of bedtime to enhance the effects of temazepam.

2. Temazepam should not be used more than three times a week for less than 3 months.

In the United States, over-the-counter drugs are regulated by: 1. No one. There is no oversight for over-the-counter medications. 2. The U.S. Food and Drug Administration Center for Drug Evaluation and Research 3. The U.S. Drug Enforcement Administration 4. MedWatch

2. The U.S. Food and Drug Administration Center for Drug Evaluation and Research

Which of the following statements about the major distribution barriers (blood-brain or fetal-placental) is true? 1. Water soluble and ionized drugs cross these barriers rapidly. 2. The blood-brain barrier slows the entry of many drugs into and from brain cells. 3. The fetal-placental barrier protects the fetus from drugs taken by the mother. 4. Lipid-soluble drugs do not pass these barriers and are safe for pregnant women.

2. The blood-brain barrier slows the entry of many drugs into and from brain cells.

When a patient is prescribed nicotine nasal spray for tobacco cessation, instructions include: 1. Inhale deeply with each dose to ensure deposition in the lungs. 2. The dose is one to two sprays in each nostril per hour, not to exceed 40 sprays per day. 3. If they have a sensation of "head rush" this indicates the medication is working well. 4. Nicotine spray may be used for up to 12 continuous months.

2. The dose is one to two sprays in each nostril per hour, not to exceed 40 sprays per day

Infants and young children are at higher risk of developing antibiotic-resistant infections due to: 1. Developmental differences in pharmacokinetics of the antibiotics in children 2. The fact that children this age are more likely to be in daycare and exposed to pathogens from other children 3. Parents of young children insisting on preventive antibiotics so they don't miss work when their child is sick 4. Immunosuppression from the multiple vaccines they receive in the first 2 years of life

2. The fact that children this age are more likely to be in daycare and exposed to pathogens from other children

Instructions for the use of nicotine gum include: 1. Chew the gum quickly to get a peak effect. 2. The gum should be "parked" in the buccal space between chewing. 3. Acidic drinks such as coffee help with the absorption of the nicotine. 4. The highest abstinence rates occur if the patient chews the gum when he or she is having cravings.

2. The gum should be "parked" in the buccal space between chewing.

The U.S. Food and Drug Administration regulates: 1. Prescribing of drugs by MDs and NPs 2. The official labeling for all prescription and over-the-counter drugs 3. Off-label recommendations for prescribing 4. Pharmaceutical educational offerings

2. The official labeling for all prescription and over-the-counter drugs

Medications are typically started for angina patients when: 1. The first permanent EKG changes occur 2. The start of class I or II symptoms 3. The events trigger a trip to the emergency department 4. When troponin levels become altered

2. The start of class I or II symptoms

Combinations of a long-acting nitrate and a beta blocker are especially effective in treating angina because: 1. Nitrates increase MOS and beta blockers increase MOD. 2. Their additive effects permit lower doses of both drugs and their adverse reactions cancel each other out. 3. They address the pathology of patients with exertional angina who have fixed atherosclerotic coronary heart disease. 4. All of the above

2. Their additive effects permit lower doses of both drugs and their adverse reactions cancel each other out.

While patient education about their drugs is important, information alone does not necessarily lead to adherence to a drug regimen. Patients report greater adherence when: 1. The provider spent a lot of time discussing the drugs with them 2. Their concerns and specific area of knowledge deficit were addressed 3. They were given written material, such as pamphlets, about the drugs 4. The provider used appropriate medical and pharmacological terms

2. Their concerns and specific area of knowledge deficit were addressed

What "onset of action" symptoms should be reviewed with patients who have been newly prescribed a selective serotonin reuptake inhibitor? 1. They will have insomnia for a week. 2. They can feel a bit of nausea, but this resolves in a week. 3. They will have an "onset seizure" but this is considered normal. 4. They will no longer dream.

2. They can feel a bit of nausea, but this resolves in a week.

In teaching about the use of sublingual nitroglycerine, the patient should be instructed: 1. To swallow the tablet with a full glass of water 2. To place one tablet under the tongue if chest pain occurs and allow it to dissolve 3. To take one tablet every 5 minutes until the chest pain goes away 4. That it should "burn" when placed under the tongue or it is no longer effective

2. To place one tablet under the tongue if chest pain occurs and allow it to dissolve

The overall goal of treating hyperlipidemia is: 1. Maintain an LDL level of less than 160 mg/dL 2. To reduce atherogenesis 3. Lowering apo B, one of the apoliproteins 4. All of the above

2. To reduce atherogenesis

The major reason for not crushing a sustained-release capsule is that, if crushed, the coated beads of the drugs could possibly result in: 1. Disintegration 2. Toxicity 3. Malabsorption 4. Deterioration

2. Toxicity

Diagnosis of heart failure cannot be made by symptoms alone because many disorders share the same symptoms. The most specific and sensitive diagnostic test for heart failure is: 1. Chest x-rays that show cephalization and measure heart size 2. Two-dimensional echocardiograms that identify structural anomalies and cardiac dysfunction 3. Complete blood count, blood urea nitrogen, and serum electrolytes that facilitate staging for end-organ damage 4. Measurement of brain natriuretic peptide to distinguish between systolic and diastolic dysfunction

2. Two-dimensional echocardiograms that identify structural anomalies and cardiac dysfunction

James has hypothalamic-pituitary-adrenal axis suppression from chronic prednisone (a corticosteroid) use. He is at risk for what type of adverse drug reaction? 1. Type B 2. Type C 3. Type E 4. Type F

2. Type C

William is a 62-year-old male who is requesting a prescription for sildenafil (Viagra). He should be screened for ________________ before prescribing sildenafil. 1. Renal dysfunction 2. Unstable coronary artery disease 3. Benign prostatic hypertrophy 4. History of priapism

2. Unstable coronary artery disease

Hispanic native healers (curanderas): 1. Are not heavily utilized by Hispanics who immigrate to the United States 2. Use herbs and teas in their treatment of illness 3. Provide unsafe advice to Hispanics and should not be trusted 4. Need to be licensed in their home country in order to practice in the United States

2. Use herbs and teas in their treatment of illness

Precautions that should be taken when prescribing controlled substances include: 1. Faxing the prescription for a Schedule II drug directly to the pharmacy 2. Using tamper-proof paper for all prescriptions written for controlled drugs 3. Keeping any pre-signed prescription pads in a locked drawer in the clinic 4. Using only numbers to indicate the amount of drug to be prescribed

2. Using tamper-proof paper for all prescriptions written for controlled drugs

Success rates for smoking cessation using NRT: 1. Are about the same regardless of the method chosen 2. Vary from 40% to 50% at 12 months 3. Both 1 and 2 4. Neither 1 nor 2

2. Vary from 40% to 50% at 12 months

Activation of central alpha2 receptors results in inhibition of cardioacceleration and ______________ centers in the brain. 1. Vasodilation 2. Vasoconstriction 3. Cardiovascular 4. Respiratory

2. Vasoconstriction

Which of the following vitamin or mineral supplements may by teratogenic if a pregnant woman takes more than the recommended amount? 1. Iron 2. Vitamin A 3. Vitamin B6 4. Vitamin C

2. Vitamin A

Steady state is: 1. The point on the drug concentration curve when absorption exceeds excretion 2. When the amount of drug in the body remains constant 3. When the amount of drug in the body stays below the minimum toxic concentration 4. All of the above

2. When the amount of drug in the body remains constant

In deciding which of multiple drugs used to use to treat a condition, the NP chooses Drug A because it: 1. Has serious side effects and it is not being used for a life-threatening condition 2. Will be taken twice daily and will be taken at home 3. Is expensive, but covered by health insurance 4. None of these are important in choosing a drug

2. Will be taken twice daily and will be taken at home

Jake, a 45-year-old patient with schizophrenia, was recently hospitalized for acute psychosis due to medication noncompliance. He was treated with IM long-acting haloperidol. Besides monitoring his schizophrenia symptoms, the patient should be assessed by his primary care provider: 1. For excessive weight loss 2. With the Abnormal Involuntary Movement Scale (AIMS) for extrapyramidal symptoms (EPS) 3. Monthly for tolerance to the haloperidol 4. Only by the mental health provider, as most NPs in primary care do not care for mentally ill patients

2. With the Abnormal Involuntary Movement Scale (AIMS) for extrapyramidal symptoms (EPS)

The type of adverse drug reaction that is idiosyncratic when a drug given in the usual therapeutic doses is type: 1. A 2. B 3. C 4. D

2.B

Digoxin may cause a type A adverse drug reaction due to: 1. Idiosyncratic effects 2. Its narrow therapeutic index 3. Being a teratogen 4. Being a carcinogen

2.Its narrow therapeutic index

Infants and young children are at higher risk of ADRs due to: 1. Immature renal function in school-age children 2. Lack of safety and efficacy studies in the pediatric population 3. Children's skin being thicker than adults, requiring higher dosages of topical medication 4. Infant boys having a higher proportion of muscle mass, leading to a higher volume of distribution

2.Lack of safety and efficacy studies in the pediatric population

Ongoing monitoring is essential after treating for gonorrhea. The patient should be rescreened for gonorrhea and chlamydia in: 1. 4 weeks 2. 3 to 6 weeks 3. 3 to 6 months 4. 1 year

3. 3 to 6 months

The American Dietetic Association recommends pregnant women take a supplement including: 1. 1,000 IU daily of vitamin D 2. 2.4 mcg/day of vitamin B12 3. 600 mcg/day of folic acid 4. 8 mg/day of iron

3. 600 mcg/day of folic acid

Which of the following patients would be at higher risk of experiencing adverse drug reactions (ADRs): 1. A 32-year-old male 2. A 22-year-old female 3. A 3-month-old female 4. A 48-year-old male

3. A 3-month-old female

When obtaining a drug history from Harold, he gives you a complete list of his prescription medications. He denies taking any other drugs, but you find that he occasionally takes aspirin for his arthritis flare ups. This is an example of: 1. His appropriately only telling you about his regularly prescribed medications 2. His hiding information regarding his inappropriate use of aspirin from you 3. A common misconception that intermittently taken over-the counter medications are not an important part of his drug history 4. A common misuse of over-the-counter aspirin

3. A common misconception that intermittently taken over-the counter medications are not an important part of his drug history

After H. pylori treatment is completed, the next step in peptic ulcer disease therapy is: 1. Testing for H. pylori eradication with a serum ELISA test 2. Endoscopy by a specialist 3. A proton pump inhibitor for 8 to 12 weeks until healing is complete 4. All of the above

3. A proton pump inhibitor for 8 to 12 weeks until healing is complete

A 56-year-old woman is complaining of vaginal dryness and dyspareunia. To treat her symptoms with the lowest adverse effects she should be prescribed: 1. Low-dose oral estrogen 2. A low-dose estrogen/progesterone combination 3. A vaginal estradiol ring 4. Vaginal progesterone cream

3. A vaginal estradiol ring

Heart failure is a leading cause of death and hospitalization in older adults (greater than 65 years old). The drug of choice for this population is: 1. Aldosterone antagonists 2. Eplerenone 3. ACE inhibitors 4. ARBs

3. ACE inhibitors

Patients who have angina, regardless of class, who are also diabetic, should be on: 1. Nitrates 2. Beta blockers 3. ACE inhibitors 4. Calcium channel blockers

3. ACE inhibitors

Angiotensin-converting-enzyme (ACE) inhibitors are a central part of the treatment of heart failure because they have more than one action to address the pathological changes in this disorder. Which of the following pathological changes in heart failure is NOT addressed by ACE inhibitors? 1. Changes in the structure of the left ventricle so that it dilates, hypertrophies, and uses energy less efficiently. 2. Reduced formation of cross-bridges so that contractile force decreases. 3. Activation of the sympathetic nervous system that increases heart rate and preload. 4. Decreased renal blood flow that decreases oxygen supply to the kidneys.

3. Activation of the sympathetic nervous system that increases heart rate and preload.

Lifestyle modifications for patients with prehypertension or hypertension include: 1. Diet and increase exercise to achieve a BMI greater than 25. 2. Drink 4 ounces of red wine at least once per week. 3. Adopt the dietary approaches to stop hypertension (DASH) diet. 4. Increase potassium intake.

3. Adopt the dietary approaches to stop hypertension (DASH) diet.

Alpha-beta blockers are especially effective to treat hypertension for which ethnic group? 1. White 2. Asian 3. African American 4. Native American

3. African American

Which of the following should not be taken with a selective serotonin reuptake inhibitor? 1. Aged blue cheese 2. Grapefruit 3. Alcohol 4. Green leafy vegetables

3. Alcohol

In choosing a benzodiazepam to treat anxiety the prescriber needs to be aware of the possibility of dependence. The benzodiazepam with the greatest likelihood of rapidly developing dependence is: 1. Chlordiazepoxide (Librium) 2. Clonazepam (Klonopin) 3. Alprazolam (Xanax) 4. Oxazepam (Serax)

3. Alprazolam (Xanax)

Food in the gastrointestinal tract affects drug absorption by: 1. Altering the pH of the colon, which decreases absorption 2. Competing with the drug for plasma proteins 3. Altering gastric emptying time 4. Altering the pH of urine

3. Altering gastric emptying time

Upregulation or hypersensitization may lead to: 1. Increased response to a drug 2. Decreased response to a drug 3. An exaggerated response if the drug is withdrawn 4. Refractoriness or complete lack of response

3. An exaggerated response if the drug is withdrawn

Women who have migraines with an aura should not be prescribed estrogen because of: 1. The interaction between triptans and estrogen, limiting migraine therapy choices 2. An increased incidence of migraines with the use of estrogen 3. An increased risk of stroke occurring with estrogen use 4. Patients with migraines may be prescribed estrogen without any concerns

3. An increased risk of stroke occurring with estrogen use

Angina is produced by an imbalance between myocardial oxygen supply (MOS) and demand (MOD) in the myocardium. Which of the following drugs help to correct this imbalance by increasing MOS? 1. Calcium channel blockers 2. Beta blockers 3. Angiotensin-converting-enzyme (ACE) inhibitors 4. Aspirin

3. Angiotensin-converting-enzyme (ACE) inhibitors

Infants with reflux are initially treated with: 1. Histamine2 receptor antagonist (ranitidine) 2. Proton pump inhibitor (omeprazole) 3. Anti-reflux maneuvers (elevate head of bed) 4. Prokinetic (metoclopramide)

3. Anti-reflux maneuvers (elevate head of bed)

Drugs that have a significant first pass effect: 1. Must be given by the enteral (oral) route only 2. Bypass the hepatic circulation 3. Are rapidly metabolized by the liver and may have little if any desired action 4. Are converted by the liver to more active and fat-soluble forms

3. Are rapidly metabolized by the liver and may have little if any desired action

Evidence is strong that the timing of HF interventions are best initiated when: 1. The person enters stage C 2. The person has functional disabilities 3. At the earliest indication 4. When stage IV is determined

3. At the earliest indication

Patient teaching related to amlodipine includes: 1. Increase calcium intake to prevent osteoporosis from a calcium blockade. 2. Do not crush the tablet; it must be given in liquid form if the patient has trouble swallowing it. 3. Avoid grapefruit juice as it affects the metabolism of this drug. 4. Rise slowly from a supine position to reduce orthostatic hypotension.

3. Avoid grapefruit juice as it affects the metabolism of this drug.

ACE inhibitors are contraindicated in pregnancy. While treatment of heart failure during pregnancy is best done by a specialist, which of the following drug classes is considered to be safe, at least in the later parts of pregnancy? 1. Diuretics 2. ARBs 3. Beta blockers 4. Nitrates

3. Beta blockers

Pharmacokinetic factors that affect prescribing include: 1. Therapeutic index 2. Minimum effective concentration 3. Bioavailability 4. Ease of titration

3. Bioavailability

Six-year-old Lucy has recently been started on ethosuximide (Zarontin) for seizures. She should be monitored for: 1. Increased seizure activity, as this drug may auto-induce seizures 2. Altered renal function, including renal failure 3. Blood dyscrasias, which are uncommon but possible 4. Central nervous system excitement, leading to insomnia

3. Blood dyscrasias, which are uncommon but possible

Abrupt withdrawal of beta blockers can be life threatening. Patients at highest risk for serious consequences of rapid withdrawal are those with: 1. Angina 2. Coronary artery disease 3. Both 1 and 2 4. Neither 1 nor 2

3. Both 1 and 2

Clonidine has several off-label uses, including: 1. Alcohol and nicotine withdrawal 2. Post-herpetic neuralgia 3. Both 1 and 2 4. Neither 1 nor 2

3. Both 1 and 2

Drugs that use CYP 3A4 isoenzymes for metabolism may: 1. Induce the metabolism of another drug 2. Inhibit the metabolism of another drug 3. Both 1 and 2 4. Neither 1 nor 2

3. Both 1 and 2

Nitrates are especially helpful for patients with angina who also have: 1. Heart failure 2. Hypertension 3. Both 1 and 2 4. Neither 1 nor 2

3. Both 1 and 2

Stage B patients should have beta blockers added to their heart failure treatment regimen when: 1. They have an ejection fraction less than 40% 2. They have had a recent MI 3. Both 1 and 2 4. Neither 1 nor 2

3. Both 1 and 2

Which of the following factors may adversely affect a patient's adherence to a therapeutic drug regimen? 1. Complexity of the drug regimen 2. Patient perception of the potential adverse effects of the drugs 3. Both 1 and 2 4. Neither 1 nor 2

3. Both 1 and 2

While taking an angiotensin II receptor blocker (ARB), patients need to avoid certain over-the-counter drugs without first consulting the provider because: 1. Cimetidine is metabolized by the CYP 3A4 isoenzymes 2. Nonsteroidal anti-inflammatory drugs reduce prostaglandin levels 3. Both 1 and 2 4. Neither 1 nor 2

3. Both 1 and 2

There is often cross-sensitivity and cross-resistance between penicillins and cephalosporins because: 1. Renal excretion is similar in both classes of drugs. 2. When these drug classes are metabolized in the liver they both produce resistant enzymes. 3. Both drug classes contain a beta-lactam ring that is vulnerable to beta-lactamase-producing organisms. 4. There is not an issue with cross-resistance between the penicillins and cephalosporins.

3. Both drug classes contain a beta-lactam ring that is vulnerable to beta-lactamase-producing organisms.

Which of the following disease processes could be made worse by taking a nonselective beta blocker? 1. Asthma 2. Diabetes 3. Both might worsen 4. Beta blockade does not affect these disorders

3. Both might worsen

Nicotine gum products are: 1. Chewed to release the nicotine and then swallowed for a systemic effect 2. "Parked" in the buccal area of the mouth to produce a constant amount of nicotine release 3. Bound to exchange resins so the nicotine is only released during chewing 4. Approximately the same in nicotine content as smoking two cigarettes

3. Bound to exchange resins so the nicotine is only released during chewing

An appropriate first-line drug to try for mild to moderate generalized anxiety disorder would be: 1. Alprazolam (Xanax) 2. Diazepam (Valium) 3. Buspirone (Buspar) 4. Amitriptyline (Elavil)

3. Buspirone (Buspar)

The American Heart Association and the American College of Cardiology have devised a classification system for heart failure that can be used to direct treatment. Patients with symptoms and underlying disease are classified as stage: 1. A 2. B 3. C 4. D

3. C

Which of the following classes of drugs is contraindicated in heart failure? 1. Nitrates 2. Long-acting dihydropyridines 3. Calcium channel blockers 4. Alpha-beta blockers

3. Calcium channel blockers

Long-term use of androgens requires specific laboratory monitoring of: 1. Glucose, calcium, testosterone, and thyroid function 2. Calcium, testosterone, PSA, and liver function 3. Calcium, testosterone, PSA, liver function, glucose, and lipids 4. CBC, testosterone, PSA, and thyroid level

3. Calcium, testosterone, PSA, liver function, glucose, and lipids

Fasting for an extended period can: 1. Increase drug absorption due to lack of competition between food and the drug 2. Alter the pH of the gastrointestinal tract, affecting absorption 3. Cause vasoconstriction, leading to decreased drug absorption 4. Shrink the stomach, causing decreased surface area for drug absorption

3. Cause vasoconstriction, leading to decreased drug absorption

Drugs are metabolized mainly by the liver via phase I or phase II reactions. The purpose of both of these types of reactions is to: 1. Inactivate prodrugs before they can be activated by target tissues 2. Change the drugs so they can cross plasma membranes 3. Change drug molecules to a form that an excretory organ can excrete 4. Make these drugs more ionized and polar to facilitate excretion

3. Change drug molecules to a form that an excretory organ can excrete

Patients at high risk for developing significant coronary heart disease are those with: 1. LDL values between 100 and 130 2. Systolic blood pressure between 120 and 130 3. Class III angina 4. Obesity

3. Class III angina

Phil is a 54-year-old male with multiple risk factors who has been on a high-dose statin for 3 months to treat his high LDL level. His LDL is 135 mg/dL and his triglycerides are elevated. A reasonable change in therapy would be to: 1. Discontinue the statin and change to a fibric acid derivative. 2. Discontinue the statin and change to ezetimibe. 3. Continue the statin and add in ezetimibe. 4. Refer him to a specialist in managing patients with recalcitrant hyperlipidemia.

3. Continue the statin and add in ezetimibe.

When considering which cholesterol-lowering drug to prescribe, which factor determines the type and intensity of treatment? 1. Total LDL 2. Fasting HDL 3. Coronary artery disease risk level 4. Fasting total cholesterol

3. Coronary artery disease risk level

The medroxyprogesterone (Depo Provera) injection has a Black Box Warning due to: 1. The potential development of significant hypertension 2. Increased risk of strokes 3. Decreased bone density 4. The risk of a life-threatening rash such as Stevens-Johnson

3. Decreased bone density

Ray has been diagnosed with hypertension and an angiotensin-converting enzyme inhibitor is determined to be needed. Prior to prescribing this drug, the NP should assess for: 1. Hypokalemia 2. Impotence 3. Decreased renal function 4. Inability to concentrate

3. Decreased renal function

Nicotine replacement therapy (NRT): 1. Is widely distributed in the body only when the gum products are used 2. Does not cross the placenta and so is safe for pregnant women 3. Delays healing of esophagitis and peptic ulcers 4. Has no drug interactions when a transdermal patch is used

3. Delays healing of esophagitis and peptic ulcers

Because primary hypertension has no identifiable cause, treatment is based on interfering with the physiological mechanisms that regulate blood pressure. Thiazide diuretics treat hypertension because they: 1. Increase renin secretion 2. Decrease the production of aldosterone 3. Deplete body sodium and reduce fluid volume 4. Decrease blood viscosity

3. Deplete body sodium and reduce fluid volume

Carbamazepine has a Black Box Warning due to life-threatening: 1. Renal toxicity, leading to renal failure 2. Hepatotoxicity, leading to liver failure 3. Dermatologic reaction, including Steven's Johnson and toxic epidermal necrolysis 4. Cardiac effects, including supraventricular tachycardia

3. Dermatologic reaction, including Steven's Johnson and toxic epidermal necrolysis

Drugs that are receptor agonists may demonstrate what property? 1. Irreversible binding to the drug receptor site 2. Up-regulation with chronic use 3. Desensitization or down-regulation with continuous use 4. Inverse relationship between drug concentration and drug action

3. Desensitization or down-regulation with continuous use

The first step in the prescribing process according to the World Health Organization is: 1. Choosing the treatment 2. Educating the patient about the medication 3. Diagnosing the patient's problem 4. Starting the treatment

3. Diagnosing the patient's problem

One major drug used to treat bipolar disease is lithium. Because lithium has a narrow therapeutic range, it is important to recognize symptoms of toxicity, such as: 1. Orthostatic hypotension 2. Agitation and irritability 3. Drowsiness and nausea 4. Painful urination and abdominal distention

3. Drowsiness and nausea

Adherence to beta blocker therapy may be affected by their: 1. Short half-lives requiring twice daily dosing 2. Tendency to elevate lipid levels 3. Effects on the male genitalia, which may produce impotence 4. None of the above

3. Effects on the male genitalia, which may produce impotence

Keng has chronic hepatitis that has led to mildly impaired liver function. He has an infection that would be best treated by a macrolide. Which would be the best choice for a patient with liver dysfunction? 1. Azithromycin (Zithromax) 2. Clarithromycin (Biaxin) 3. Erythromycin (E-mycin) 4. None of the above

3. Erythromycin (E-mycin)

Postmenopausal women with an intact uterus should not be prescribed: 1. Estrogen/progesterone combination 2. IM medroxyprogesterone (Depo Provera) 3. Estrogen alone 4. Androgens

3. Estrogen alone

Which of the following drugs has been associated with increased risk for myocardial infarction in women? 1. Aspirin 2. Beta blockers 3. Estrogen replacement 4. Lipid-lowering agents

3. Estrogen replacement

Nicole is a 16-year-old female who is taking minocycline for acne. She comes to the clinic complaining of a headache. What would be the plan of care? 1. Advise acetaminophen or ibuprofen as needed for headaches. 2. Prescribe sumatriptan (Imitrex) to be taken at the onset of the headache. 3. Evaluate her for pseudotremor cerebri. 4. Assess her caffeine intake and sleep patterns.

3. Evaluate her for pseudotremor cerebri.

Han is a 48-year-old diabetic with hyperlipidemia and high triglycerides. His LDL is 112 mg/dL and he has not tolerated statins. He warrants a trial of a: 1. Sterol 2. Niacin 3. Fibric acid derivative 4. Bile acid-binding resin

3. Fibric acid derivative

What educational points concerning fluid intake must be covered with diuretic prescriptions? 1. Fluid should be restricted when on them. 2. Fluids should contain at least one salty item daily. 3. Fluid intake should remain near normal for optimal performance. 4. Avoidance of potassium-rich fluids is encouraged.

3. Fluid intake should remain near normal for optimal performance.

A patient with a new onset of systolic ejection murmur should be assessed for which nutritional deficiency? 1. Vitamin B12 2. Vitamin C 3. Folate 4. Niacine

3. Folate

Cost of antianginal drug therapy should be considered in drug selection because of all of the following EXCEPT: 1. Patients often require multiple drugs 2. A large number of angina patients are older adults on fixed incomes 3. Generic formulations may be cheaper but are rarely bioequivalent 4. Lack of drug selectivity may result in increased adverse reactions

3. Generic formulations may be cheaper but are rarely bioequivalent

Symptoms of folate deficiency include: 1. Thinning of the hair 2. Bruising easily 3. Glossitis 4. Numbness and tingling of the hands and feet

3. Glossitis

Nonadherence is especially common in drugs that treat asymptomatic conditions, such as hypertension. One way to reduce the likelihood of nonadherence to these drugs is to prescribe a drug that: 1. Has a short half-life so that missing one dose has limited effect 2. Requires several dosage titrations so that missed doses can be replaced with lower doses to keep costs down 3. Has a tolerability profile with fewer of the adverse effects that are considered "irritating," such as nausea and dizziness 4. Must be taken no more than twice a day

3. Has a tolerability profile with fewer of the adverse effects that are considered "irritating," such as nausea and dizziness

Omega 3 fatty acids are best used to help treat: 1. High HDL 2. Low LDL 3. High triglycerides 4. Any high lipid value

3. High triglycerides

Depo Provera is prescribed intramuscularly to create a storage reservoir of the drug. Storage reservoirs: 1. Assure that the drug will reach its intended target tissue 2. Are the reason for giving loading doses 3. Increase the length of time a drug is available and active 4. Are most common in collagen tissues

3. Increase the length of time a drug is available and active

Metoclopramide improves gastroesophageal reflux disease symptoms by: 1. Reducing acid secretion 2. Increasing gastric pH 3. Increasing lower esophageal tone 4. Decreasing lower esophageal tone

3. Increasing lower esophageal tone

Cruciferous vegetables may alter drug pharmacokinetics by: 1. Enhancing absorption of weakly acidic drugs 2. Altering CYP 3A4 activity, leading to elevated levels of drugs, such as the statins 3. Inducing CYP 1A2, possibly leading to therapeutic failure of drugs metabolized by CYP 1A2 4. Decreasing first-pass metabolism of drugs

3. Inducing CYP 1A2, possibly leading to therapeutic failure of drugs metabolized by CYP 1A2

Phenytoin decreases folic acid absorption by: 1. Altering the pH of the stomach 2. Increasing gastric emptying time 3. Inhibiting intestinal enzymes required for folic acid absorption 4. Chelation of the folic acid into inactive ingredients

3. Inhibiting intestinal enzymes required for folic acid absorption

Drugs that are prone to cause adverse drug effects include: 1. Diuretics 2. Inhaled anticholinergics 3. Insulins 4. Stimulants

3. Insulins

Antacids such as calcium carbonate (Tums) can reduce the absorption of which of the following nutrients? 1. Protein 2. Calcium 3. Iron 4. Vitamin K

3. Iron

Women who are prescribed progestin-only contraception need education regarding which common adverse drug effects? 1. Increased migraine headaches 2. Increased risk of developing blood clots 3. Irregular vaginal bleeding for the first few months 4. Increased risk for hypercalcemia

3. Irregular vaginal bleeding for the first few months

A 24-year-old male received multiple fractures in a motor vehicle accident that required significant amounts of opioid medication to treat his pain. He is at risk for a _____ adverse drug reaction when he no longer requires the opioids. 1. Rapid 2. First-dose 3. Late 4. Delayed

3. Late

Off-label prescribing is: 1. Regulated by the U.S. Food and Drug Administration 2. Illegal by NPs in all states (provinces) 3. Legal if there is scientific evidence for the use 4. Regulated by the Drug Enforcement Administration

3. Legal if there is scientific evidence for the use

Erroneous information about LGBTQ individuals can lead to failure to give accurate advice to them as patients. Which of the following statements is true about lesbians: 1. Lesbians cannot contract a sexually transmitted infection from their female partner. 2. Screening for cervical cancer is not required. 3. Lesbians as a group are less likely to have health-care insurance. 4. Like women in general, lesbians are more likely than gay men to seek care for health-related issues.

3. Lesbians as a group are less likely to have health-care insurance.

First-line therapy for hyperlipidemia is: 1. Statins 2. Niacin 3. Lifestyle changes 4. Bile acid-binding resins

3. Lifestyle changes

Factors that affect gastric drug absorption include 1. Liver enzyme activity 2. Protein-binding properties of the drug molecule 3. Lipid solubility of the drug 4. Ability to chew and swallow

3. Lipid solubility of the drug

Fiber supplements are great options for elderly patients who have the concurrent problem of: 1. End-stage renal failure on fluid restriction 2. Recurrent episodes of diarrhea several times a day 3. Long-term issues of constipation 4. Needing to take multiple medications around the clock every 2 hours

3. Long-term issues of constipation

When discussing with a patient the different start methods used for oral combined contraceptives, the advantage of a Sunday start over the other start methods is: 1. Immediate protection against pregnancy the first week of using the pill 2. No back-up method is needed when starting 3. Menses occur during the week 4. They can start the pill on the Sunday after the office visit

3. Menses occur during the week

Colestipol comes in a powdered form. The patient is taught to: 1. Take the powder dry and follow it with at least 8 ounces of water 2. Take it with a meal to enhance its action on fatty food 3. Mix the powder with 4 to 6 ounces of milk or fruit juice 4. Take after the evening meal to coincide with cholesterol synthesis

3. Mix the powder with 4 to 6 ounces of milk or fruit juice

To reduce potential adverse effects, patients taking a peripherally acting alpha1 antagonist should do all of the following EXCEPT: 1. Take the dose at bedtime 2. Sit up slowly and dangle their feet before standing 3. Monitor their blood pressure and skip a dose if the pressure is less than 120/80 4. Weigh daily and report weight gain of greater than 2 pounds in one day

3. Monitor their blood pressure and skip a dose if the pressure is less than 120/80

According to the National Standards of Culturally and Linguistically Appropriate Services, an interpreter for health care: 1. May be a bilingual family member 2. May be a bilingual nurse or other health-care provider 3. Must be a professionally trained medical interpreter 4. Must be an employee of the organization

3. Must be a professionally trained medical interpreter

The interpretation of DEXA scores in the rare cases of adolescent osteoporosis in teens: 1. Use the same T scores that are established for women 2. Cannot be done because of less-than-mature bones 3. Must use special Z-scores developed for this reason 4. Can only be done if bisphosphonates have already been started

3. Must use special Z-scores developed for this reason

Disease states in addition to hypertension in which beta blockade is a compelling indication for the use of beta blockers include: 1. Heart failure 2. Angina 3. Myocardial infarction 4. Dyslipidemia

3. Myocardial infarction

Dysmenorrhea is one of the most common gynecological complaints in young women. The first line of drug treatment for this disorder is: 1. Oral contraceptive pills 2. Caffeine 3. NSAIDs 4. Aspirin

3. NSAIDs

An ACE inhibitor and what other class of drug may reduce proteinuria in patients with diabetes better than either drug alone? 1. Beta blockers 2. Diuretics 3. Nondihydropyridine calcium channel blockers 4. Angiotensin II receptor blockers

3. Nondihydropyridine calcium channel blockers

When prescribing a tetracycline or quinolone antibiotic it is critical to instruct the patient: 1. Not to take their regularly prescribed medications while on these antibiotics 2. Regarding the need for lots of acidic foods and juices, such as orange juice, to enhance absorption 3. Not to take antacids while on these medications, as the antacid decreases absorption 4. That there are no drug interactions with these antibiotics

3. Not to take antacids while on these medications, as the antacid decreases absorption

The point in time on the drug concentration curve that indicates the first sign of a therapeutic effect is the: 1. Minimum adverse effect level 2. Peak of action 3. Onset of action 4. Therapeutic range

3. Onset of action

The National Standards of Culturally and Linguistically Appropriate Services are required to be implemented in all: 1. Hospitals 2. Clinics that serve the poor 3. Organizations that receive federal funds 4. Clinics that serve ethnic minorities

3. Organizations that receive federal funds

The direct costs of drug therapy include: 1. The actual cost of acquiring the medication 2. The loss of income due to illness 3. Pain and suffering due to inadequate drug therapy 4. The cost of a funeral associated with premature death

3. Pain and suffering due to inadequate drug therapy

A client asks the NP about the differences in drug effects between men and women. What is known about the differences between the pharmacokinetics of men and women? 1. Body temperature varies between men and women. 2. Muscle mass is greater in women. 3. Percentage of fat differs between genders. 4. Proven subjective factors exist between the genders.

3. Percentage of fat differs between genders.

It is reasonable to recommend supplementation with _________ in the treatment of hyperlipidemia. 1. Omega-3 fatty acids 2. Probiotics 3. Plant sterols 4. Calcium

3. Plant sterols

A test of cure is recommended after treating chlamydia in which patient population? 1. Men who have sex with men 2. Adolescent females 3. Pregnant patients 4. All of the above

3. Pregnant patients

Ashley comes to the clinic with a request for oral contraceptives. She has successfully used oral contraceptives before and has recently started dating a new boyfriend so would like to restart contraception. She denies recent intercourse and has a negative urine pregnancy test in the clinic. An appropriate plan of care would be: 1. Recommend she return to the clinic at the start of her next menses to get a Depo Provera shot. 2. Prescribe oral combined contraceptives and recommend she start them at the beginning of her next period and use a back-up method for the first 7 days. 3. Prescribe oral contraceptives and have her start them the same day as the visit with a back-up method used for the first 7 days. 4. Discuss the advantages of using the topical birth control patch and recommend she consider using the patch.

3. Prescribe oral contraceptives and have her start them the same day as the visit with a back-up method used for the first 7 days.

Actions taken to reduce drug—drug interaction problems include all of the following EXCEPT: 1. Reducing the dosage of one of the drugs 2. Scheduling their administration at different times 3. Prescribing a third drug to counteract the adverse reaction of the combination 4. Reducing the dosage of both drugs

3. Prescribing a third drug to counteract the adverse reaction of the combination

An acceptable first-line treatment for peptic ulcer disease with positive H. pylori test is: 1. Histamine2 receptor antagonists for 4 to 8 weeks 2. Proton pump inhibitor bid for 12 weeks until healing is complete 3. Proton pump inhibitor bid plus clarithromycin plus amoxicillin for 14 days 4. Proton pump inhibitor bid and levofloxacin for 14 days

3. Proton pump inhibitor bid plus clarithromycin plus amoxicillin for 14 days

Samantha is taking lamotrigine (Lamictal) for her seizures and requests a prescription for combined oral contraceptives (COCs), which interact with lamotrigine and may cause: 1. Contraceptive failure 2. Excessive weight gain 3. Reduced lamotrigine levels, requiring doubling the dose of lamotrigine 4. Induction of estrogen metabolism, requiring higher estrogen content OCs be prescribed

3. Reduced lamotrigine levels, requiring doubling the dose of lamotrigine

James tells you that he is confused by his Medicare Part D coverage plan. An appropriate intervention would be: 1. Order cognitive testing to determine the source of his confusion. 2. Sit down with him and explain the whole Medicare Part D process. 3. Refer him to the Medicare specialist in his insurance plan to explain the benefit to him. 4. Request his son come to the next appointment so you can explain the benefit to him.

3. Refer him to the Medicare specialist in his insurance plan to explain the benefit to him

The "donut hole" in Medicare Part D: 1. Will be totally eliminated with the federal health-care reform enacted in 2010 2. Refers to the period of time when annual individual drug costs are between $250 and $2,250 per year and drug costs are covered 75% 3. Refers to the period between when the annual individual drug costs are $2,970 and $4,750 and the patient pays 52.5% of the costs of brand name drugs (2013) 4. Has no effect on whether patients continue to fill their prescriptions during the coverage gap

3. Refers to the period between when the annual individual drug costs are $2,970 and $4,750 and the patient pays 52.5% of the costs of brand name drugs (2013)

Sadie is an 82-year-old patient who has herpes zoster (shingles) and would benefit from an antiviral such as valacyclovir. Prior to prescribing valacyclovir she will need an assessment of: 1. Complete blood count to rule out anemia 2. Liver function 3. Renal function 4. Immunocompetence

3. Renal function

Donald has been diagnosed with hyperlipidemia. Based on his lipid profile, atorvastatin is prescribed. Rhabdomyolysis is a rare but serious adverse response to this drug. Donald should be told to: 1. Become a vegetarian because this disorder is associated with eating red meat. 2. Stop taking the drug if abdominal cramps and diarrhea develop. 3. Report muscle weakness or tenderness and dark urine to his provider immediately. 4. Expect "hot flash" sensations during the first 2 weeks of therapy.

3. Report muscle weakness or tenderness and dark urine to his provider immediately.

Ranolazine is used in angina patients to: 1. Dilate plaque-filled arteries 2. Inhibit platelet aggregation 3. Restrict late sodium flow in the myocytes 4. Induce vasoconstriction in the periphery to open coronary vessels

3. Restrict late sodium flow in the myocytes

Taking which drug with food maximizes it bioavailability? 1. Donepezil 2. Galantamine 3. Rivastigmine 4. Memantine

3. Rivastigmine

There is strong evidence to support that adequate vitamin C intake prevents: 1. The common cold 2. Breast cancer 3. Scurvy 4. All of the above

3. Scurvy

Which of the following drug classes is associated with significant differences in metabolism based on gender? 1. Beta blockers 2. Antibiotics 3. Serotonin reuptake inhibitors 4. Angiotensin-converting-enzyme (ACE) inhibitors

3. Serotonin reuptake inhibitors

Janice has elevated LDL, VLDL, and triglyceride levels. Niaspan, an extended-release form of niacin, is chosen to treat her hyperlipidemia. Due to its metabolism and excretion, which of the following laboratory results should be monitored? 1. Serum alanine aminotransferase 2. Serum amylase 3. Serum creatinine 4. Phenylketonuria

3. Serum creatinine

Laboratory monitoring for patients on angiotensin-converting enzyme inhibitors or angiotensin II receptor blockers should include: 1. White blood cell counts with the drug dosage increased for elevations above 10,000 feet 2. Liver function tests with the drug dosage stopped for alanine aminotransferase values twice that of normal 3. Serum creatinine levels with the drug dosage reduced for values greater than 2.5 mg/dL 4. Serum glucose levels with the drug dosage increased for levels greater than 120 mg/dL

3. Serum creatinine levels with the drug dosage reduced for values greater than 2.5 mg/dL

Mary has a two-tiered prescription benefit plan, which means: 1. She can receive differing levels of care based on whether she chooses an "in-plan" provider or not. 2. She is eligible for the new Medicare Part D "donut hole" reduction of costs program. 3. She pays a higher copay for brand-name drugs than for generic drugs. 4. She must always choose to be treated with generic drugs first.

3. She pays a higher copay for brand-name drugs than for generic drugs.

A 19-year-old female is a nasal Staph aureus carrier and is placed on 5 days of rifampin for treatment. Her only other medication is combined oral contraceptives. What education should she receive regarding her medications? 1. Separate the oral ingestion of the rifampin and oral contraceptive by at least an hour. 2. Both medications are best tolerated if taken on an empty stomach. 3. She should use a back-up method of birth control such as condoms for the rest of the current pill pack. 4. If she gets nauseated with the medications she should call the office for an antiemetic prescription.

3. She should use a back-up method of birth control such as condoms for the rest of the current pill pack.

Blood pressure checks in children: 1. Should occur with their annual physical examinations after 6 years of age 2. Require a blood pressure cuff that is one-third the diameter of the child's arm 3. Should be done during every health-care visit after 3 years of age 4. Require additional laboratory tests such as serum creatinine

3. Should be done during every health-care visit after 3 years of age

The most common adverse effect of the transdermal nicotine replacement patch is: 1. Nicotine toxicity 2. Tingling at the site of patch application 3. Skin irritation under the patch site 4. Life-threatening dysrhythmias

3. Skin irritation under the patch site

James is a 45-year-old patient with an LDL level of 120 and normal triglycerides. Appropriate first-line therapy for James may include diet counseling, increased physical activity, and: 1. A statin 2. Niacin 3. Sterols 4. A fibric acid derivative

3. Sterols

Jose is a 12-year-old overweight child with a total cholesterol of 180 mg/dL and LDL of 125 mg/dL. Along with diet education and recommending increased physical activity, a treatment plan for Jose would include ____________ with a reevaluation in 6 months. 1. Statins 2. Niacin 3. Sterols 4. Bile acid-binding resins

3. Sterols

Amiodarone has been prescribed in a patient with a supraventricular dysrhythmia. Patient teaching should include all of the following EXCEPT: 1. Notify your health-care provider immediately if you have visual change. 2. Monitor your own blood pressure and pulse daily. 3. Take a hot shower or bath if you feel dizzy. 4. Use a sunscreen on exposed body surfaces.

3. Take a hot shower or bath if you feel dizzy.

Rapid-acting nitrates are important for all angina patients. Which of the following are true statements about their use? 1. These drugs are useful for immediate symptom relief when the patient is certain it is angina. 2. The dose is one sublingual tablet or spray every 5 minutes until the chest pain goes away. 3. Take one nitroglycerine tablet or spray at the first sign of angina; repeat every 5 minutes for no more than two doses. If chest pain is still not relieved, call 911. 4. All of the above

3. Take one nitroglycerine tablet or spray at the first sign of angina; repeat every 5 minutes for no more than two doses. If chest pain is still not relieved, call 911.

Fluoroquinolones have a Black Box Warning regarding ________ even months after treatment. 1. Renal dysfunction 2. Hepatic toxicity 3. Tendon rupture 4. Development of glaucoma

3. Tendon rupture

Nurse practitioner prescriptive authority is regulated by: 1. The National Council of State Boards of Nursing 2. The U.S. Drug Enforcement Administration 3. The State Board of Nursing for each state 4. The State Board of Pharmacy

3. The State Board of Nursing for each state

Which diuretic agents typically do not need potassium supplementation? 1. The loop diuretics 2. The thiazide diuretics 3. The aldosterone inhibitors 4. They all need supplementation

3. The aldosterone inhibitors

Pharmacoeconomics is: 1. The study of the part of the U.S. economy devoted to drug use 2. The study of the impact of prescription drug costs on the overall economy 3. The analysis of the costs and consequences of any health-care-related treatment or service 4. The analysis of the clinical efficacy of the drug

3. The analysis of the costs and consequences of any health-care-related treatment or service

Before prescribing phentermine to Sarah, a thorough drug history should be taken including assessing for the use of serotonergic agents such as selective serotonin reuptake inhibitors (SSRIs) and St John's wort due to: 1. Additive respiratory depression risk 2. Additive effects affecting liver function 3. The risk of serotonin syndrome 4. The risk of altered cognitive functioning

3. The risk of serotonin syndrome

What can chest x-rays contribute to the diagnosis and management of HF? 1. They have no role. 2. They can give very precise pictures of pulmonary fluid status. 3. They provide an idea of general cardiac size and pulmonary great vessel distribution. 4. They can confirm the diagnosis.

3. They provide an idea of general cardiac size and pulmonary great vessel distribution.

Elena Vasquez's primary language is Spanish, and she speaks very limited English. Which technique would be appropriate to use in teaching her about a new drug you have just prescribed? 1. Use correct medical terminology because Spanish has a Latin base. 2. Use a family member who speaks more English to act as an interpreter. 3. Use a professional interpreter or a reliable staff member who can act as an interpreter. 4. Use careful, detailed explanations.

3. Use a professional interpreter or a reliable staff member who can act as an interpreter.

Patients who choose the nicotine lozenge to assist in quitting tobacco should be instructed: 1. Chew the lozenge well. 2. Drink at least 8 ounces of water after the lozenge dissolves. 3. Use one lozenge every 1 to 2 hours (at least nine per day with a maximum of 20 per day). 4. A tingling sensation in the mouth should be reported to the provider.

3. Use one lozenge every 1 to 2 hours (at least nine per day with a maximum of 20 per day).

HF patients frequently take more than one drug. When are anticoagulants typically used? 1. When the patient enters stage III 2. Only in cases of diastolic failure 3. When there is concurrent A Fib 4. In all cases

3. When there is concurrent A Fib

Lauren is a 13-year-old child who comes to clinic with a 4-day history of cough, low-grade fever, and rhinorrhea. When she blows her nose or coughs the mucous is greenish-yellow. The appropriate antibiotic to prescribe would be: 1. Amoxicillin 2. Amoxicillin/clavulanate 3. TMP/SMZ (Septra) 4. None

4, none

Direct renin inhibitors have the following properties. They: 1. Are primarily generic drugs 2. Are a renin-angiotensin-aldosterone system (RAAS) medication that is safe during pregnancy 3. Can be used with an angiotensin-converting enzyme and angiotensin II receptor blocker medications for stronger impact 4. "Shut down" the entire RAAS cycle

4. "Shut down" the entire RAAS cycle

Instructions to a client regarding self-administration of oral enteric-coated tablets should include which of the following statements? 1. "Avoid any other oral medicines while taking this drug." 2. "If swallowing this tablet is difficult, dissolve it in 3 ounces of orange juice." 3. "The tablet may be crushed if you have any difficulty taking it." 4. "To achieve best effect, take the tablet with at least 8 ounces of fluid."

4. "To achieve best effect, take the tablet with at least 8 ounces of fluid."

ACE inhibitors are a foundational medication in HF. Which group of patients cannot take them safely? 1. Elderly patients with reduced renal clearance 2. Pregnant women 3. Women under age 30 4. 1 and 2

4. 1 and 2

Absolute contraindications that clinicians must consider when initiating estrogen therapy include: 1. Undiagnosed dysfunctional uterine bleeding 2. Deep vein or arterial thromboemboli within the prior year 3. Endometriosis 4. 1 and 2 5. All of the above

4. 1 and 2

The American Heart Association and the American Dietetic Association recommend a minimum daily fiber intake of ______ for cardiovascular health: 1. 10 mg/day 2. 15 mg/day 3. 20 mg/day 4. 25 mg/day

4. 25 mg/day

Smokers are at risk for vitamin C deficiency. It is recommended that smokers take _______ vitamin C supplement. 1. 100 mg/day 2. 500 mg/day 3. 1,000 mg/day 4. 35 mg/day more than nonsmokers

4. 35 mg/day more than nonsmokers

Effects of estrogen include: 1. Regulation of the menstrual cycle 2. Maintenance of bone density by increasing bone reabsorption 3. Maintenance of the normal structure of the skin and blood vessels 4. A and C 5. All of the above

4. A and C

Providers should use an antibiogram when prescribing. An antibiogram is: 1. The other name for the Centers for Disease Control guidelines for prescribing antibiotics 2. An algorithm used for prescribing antibiotics for certain infections 3. The reference also known as the Pink Book, published by the Centers for Disease Control 4. A chart of the local resistance patterns to antibiotics developed by laboratories

4. A chart of the local resistance patterns to antibiotics developed by laboratories

Monitoring for patients who are on long-term antifungal therapy with ketoconazole includes: 1. Platelet count 2. BUN and creatinine 3. White blood cell count 4. AST, ALT, alkaline phosphatase, and bilirubin

4. AST, ALT, alkaline phosphatase, and bilirubin

Two different pain medications are given together for pain relief. The drug—drug interaction is: 1. Synergistic 2. Antagonistic 3. Potentiative 4. Additive

4. Additive

Tetracyclines should not be prescribed to children younger than 8 years due to: 1. Risk of developing cartilage problems 2. Development of significant diarrhea 3. Risk of kernicterus 4. Adverse effects on bone growth

4. Adverse effects on bone growth

The most frequent type of drug-food interaction is food: 1. Causing increased therapeutic drug levels 2. Affecting the metabolism of the drug 3. Altering the volume of distribution of drugs 4. Affecting the gastrointestinal absorption of drugs

4. Affecting the gastrointestinal absorption of drugs

The elderly are at high risk of ADRs due to: 1. Having greater muscle mass than younger adults, leading to higher volume of distribution 2. The extensive studies that have been conducted on drug safety in this age group 3. The blood-brain barrier being less permeable, requiring higher doses to achieve therapeutic effect 4. Age-related decrease in renal function

4. Age-related decrease in renal function

A 66-year-old male was prescribed phenelzine (Nardil) while in an acute psychiatric unit for recalcitrant depression. The NP managing his primary health care needs to understand the following regarding phenelzine and other monoamine oxidase inhibitors (MAOIs): 1. He should not be prescribed any serotonergic drug such as sumatriptan (Imitrex) 2. MAOIs interact with many common foods, including yogurt, sour cream, and soy sauce 3. Symptoms of hypertensive crisis (headache, tachycardia, sweating) require immediate treatment 4. All of the above

4. All of the above

A prescription needs to be written for: 1. Legend drugs 2. Most controlled drugs 3. Medical devices 4. All of the above

4. All of the above

Adequate vitamin D is needed for: 1. Absorption of calcium from the gastrointestinal tract 2. Regulation of serum calcium levels 3. Regulation of serum phosphate levels 4. All of the above

4. All of the above

Angiotensin-converting enzyme (ACE) inhibitors treat hypertension because they: 1. Reduce sodium and water retention 2. Decrease vasoconstriction 3. Increase vasodilation 4. All of the above

4. All of the above

Angiotensin-converting enzyme inhibitors are the drug of choice in treating hypertension in diabetic patients because they: 1. Improve insulin sensitivity 2. Improve renal hemodynamics 3. Reduce the production of angiotensin II 4. All of the above

4. All of the above

Because of their longer life expectancy, women are more likely than men to experience a disabling condition. Common conditions in older women that can produce disability include: 1. Depression 2. Panic disorders 3. Dementia 4. All of the above

4. All of the above

Bismuth subsalicylate (Pepto Bismol) is a common OTC remedy for gastrointestinal complaints. Bismuth subsalicylate: 1. May lead to toxicity if taken with aspirin 2. Is contraindicated in children with flu-like illness 3. Has antimicrobial effects against bacterial and viral enteropathogens 4. All of the above

4. All of the above

Carvedilol is heavily metabolized by CYP2D6 and 2C9, resulting in drug interactions with which of the following drug classes? 1. Histamine 2 blockers 2. Quinolones 3. Serotonin re-uptake inhibitors 4. All of the above

4. All of the above

Cholinergic blockers are used to: 1. Counteract the extrapyramidal symptoms (EPS) effects of phenothiazines 2. Control tremors and relax smooth muscle in Parkinson's disease 3. Inhibit the muscarinic action of ACh on bladder muscle 4. All of the above

4. All of the above

Cultural factors that must be taken into account when prescribing include(s): 1. Who the decision maker is in the family regarding health-care decisions 2. The patient's view of health and illness 3. Attitudes regarding the use of drugs to treat illness 4. All of the above

4. All of the above

Factors in chronic conditions that contribute to nonadherence include: 1. The complexity of the treatment regimen 2. The length of time over which it must be taken 3. Breaks in the usual daily routine, such as vacations and weekends 4. All of the above

4. All of the above

If a patient is allergic to sulfonamide antibiotics, he or she will most likely have cross-sensitivity to: 1. Loop diuretics 2. Sulfonylureas 3. Thiazide diuretics 4. All of the above

4. All of the above

In addition to antimicrobial therapy, patients treated for Trichomonas infection should be educated regarding: 1. Necessity of treating sexual partner simultaneously 2. Abstaining from intercourse until both partners are treated 3. Need for retesting in 3 months due to high reinfection rate 4. All of the above

4. All of the above

Lack of adherence to blood pressure management is very common. Reasons for this lack of adherence include: 1. Lifestyle changes are difficult to achieve and maintain. 2. Adverse drug reactions are common and often fall into the categories more associated with nonadherence. 3. Costs of drugs and monitoring with laboratory tests can be expensive. 4. All of the above

4. All of the above

Men who are prescribed an erectile dysfunction drug such as sildenafil (Viagra) should be warned about the risk for: 1. Impotence when combined with antihypertensives 2. Fatal hypotension if combined with nitrates 3. Weight gain if combined with antidepressants 4. All of the above

4. All of the above

Michael asks you about why some drugs are over-the-counter and some are prescription. You explain that in order for a drug to be approved for over-the-counter use the drug must: 1. Be safe and labeled for appropriate use 2. Have a low potential for abuse or misuse 3. Be taken for a condition the patient can reliably self-diagnose 4. All of the above

4. All of the above

Monitoring adherence can take several forms, including: 1. Patient reports from data in a drug diary 2. Pill counts 3. Laboratory reports and other diagnostic markers 4. All of the above

4. All of the above

Monitoring for a child on methylphenidate for attention deficit hyperactivity disorder (ADHD) includes: 1. ADHD symptoms 2. Routine height and weight checks 3. Amount of methylphenidate being used 4. All of the above

4. All of the above

Nicotine replacement therapy should not be used in which patients? 1. Pregnant women 2. Patients with worsening angina pectoris 3. Patients who have just suffered an acute myocardial infarction 4. All of the above

4. All of the above

Nicotine withdrawal symptoms include: 1. Nervousness 2. Increased appetite 3. Difficulty concentrating 4. All of the above

4. All of the above

Not all chest pain is caused by myocardial ischemia. Noncardiac causes of chest pain include: 1. Pulmonary embolism 2. Pneumonia 3. Gastroesophageal reflux 4. All of the above

4. All of the above

Once they have been metabolized by the liver, the metabolites may be: 1. More active than the parent drug 2. Less active than the parent drug 3. Totally "deactivated" so they are excreted without any effect 4. All of the above

4. All of the above

Patients taking antacids should be educated regarding these drugs, including letting them know that: 1. They may cause constipation or diarrhea 2. Many are high in sodium 3. They should separate antacids from other medications by 1 hour 4. All of the above

4. All of the above

Progesterone-only pills are recommended for women who: 1. Are breastfeeding 2. Have a history of migraine 3. Have a medical history that contradicts the use of estrogen 4. All of the above

4. All of the above

Situations that suggest referral to a specialist is appropriate include: 1. When chronic stable angina becomes unpredictable in its characteristics and precipitating factors 2. When a post-myocardial infarction patient develops new-onset angina 3. When standard therapy is not successful in improving exercise tolerance or reducing the incidence of angina 4. All of the above

4. All of the above

Strategies prescribers can use to prevent misuse of controlled prescription drugs include: 1. Use of chemical dependency screening tools 2. Firm limit-setting regarding prescribing controlled substances 3. Practicing "just say no" to deal with patients who are pushing the provider to prescribe controlled substances 4. All of the above

4. All of the above

Tetracycline needs to be given on an empty stomach because it chelates with: 1. Calcium 2. Magnesium 3. Iron 4. All of the above

4. All of the above

The goals of treatment when prescribing for sexually transmitted infections include: 1. Treatment of infection 2. Prevention of disease spread 3. Prevention of long-term sequelae from the infection 4. All of the above

4. All of the above

The therapeutic goals when prescribing include(s): 1. Curative 2. Palliative 3. Preventive 4. All of the above

4. All of the above

When prescribing an opioid analgesic such as acetaminophen and codeine (Tylenol #3), instructions to the patient should include: 1. The medication may cause sedation and they should not drive. 2. Constipation is a common side effect and they should increase fluids and fiber. 3. Patients should not take any other acetaminophen-containing medications at the same time. 4. All of the above

4. All of the above

When starting a patient on a statin, education would include: 1. If they stop the medication their lipid levels will return to pretreatment levels. 2. Medication is a supplement to diet therapy and exercise. 3. If they have any muscle aches or pain, they should contact their provider. 4. All of the above

4. All of the above

Food can alter the pH of the stomach, leading to: 1. Enhanced drug metabolism 2. Altered vitamin K absorption 3. Increased vitamin D absorption 4. Altered drug bioavailability

4. Altered drug bioavailability

The U.S. Food and Drug Administration MedWatch system is activated when: 1. There is an adverse event to a vaccine. 2. The patient has a severe reaction that is noted in the "Severe Reaction" section in the medication label. 3. A lactating woman takes a medication that is potentially toxic to the breastfeeding infant. 4. An adverse event or serious problem occurs with a medication that is not already identified on the label.

4. An adverse event or serious problem occurs with a medication that is not already identified on the label.

Probiotics are recommended to be co-administered when ____________ are prescribed: 1. Antacids 2. Antihypertensives 3. Antidiarrheals 4. Antibiotics

4. Antibiotics

Heart failure is a chronic condition that can be adequately managed in primary care. However, consultation with or referral to a cardiologist is appropriate when: 1. Symptoms markedly worsen or the patient becomes hypotensive and has syncope 2. There is evidence of progressive renal insufficiency or failure 3. The patient remains symptomatic on optimal doses of an ACE inhibitor, a beta blocker, and a diuretic 4. Any of the above

4. Any of the above

Sydney presents to the clinic with vulvovaginal candidiasis. Appropriate treatment for her would be: 1. OTC intravaginal clotrimazole 2. OTC intravaginal miconazole 3. Oral fluconazole one-time dose 4. Any of the above

4. Any of the above

Treatment for chancroid in a nonpregnant patient would be: 1. Oral azithromycin 2. IM ceftriaxone 3. Oral ciprofloxacin 4. Any of the above

4. Any of the above

Jonathan has been diagnosed with strep throat and needs a prescription for an antibiotic. He says the last time he had penicillin he developed a red, blotchy rash. An appropriate antibiotic to prescribe would be: 1. Penicillin VK, because his rash does not sound like a serious rash 2. Amoxicillin 3. Cefadroxil (Duricef) 4. Azithromycin

4. Azithromycin

A contraindication to the use of combined contraceptives is: 1. Adolescence (not approved for this age) 2. A history of clotting disorder 3. Recent pregnancy 4. Being overweight

4. Being overweight

A patient presents with a complaint of dark stools and epigastric pain described as gnawing and burning. Which of the medications is the most likely cause? 1. Acetaminophen 2. Estradiol 3. Donepezil 4. Bethanechol

4. Bethanechol

Jamie is a 34-year-old pregnant woman with familial hyperlipidemia and elevated LDL levels. What is the appropriate treatment for a pregnant woman? 1. A statin 2. Niacin 3. Fibric acid derivative 4. Bile acid-binding resins

4. Bile acid-binding resins

Angiotensin-converting enzyme inhibitors are useful in a variety of disorders. Which of the following statements are true about both its usefulness in the disorder and the reason for its use? 1. Stable angina because it decreases the thickening of vascular walls due to decreased modified release. 2. Heart failure because it reduces remodeling of injured myocardial tissues. 3. Both 1 and 2 are true and the reasons are correct. 4. Both 1 and 2 are true but the reasons are wrong. 5. Neither 1 nor 2 are true.

4. Both 1 and 2 are true but the reasons are wrong.

A patient with anxiety and depression may respond to: 1. Duloxetine (Cymbalta) 2. Fluoxetine (Prozac) 3. Oxazepam (Serax) 4. Buspirone (Buspar) and an SSRI combined

4. Buspirone (Buspar) and an SSRI combined

Obese women may have increased risk of failure with which contraceptive method? 1. Combined oral contraceptives 2. Progestin-only oral contraceptive pill 3. Injectable progestin 4. Combined topical patch

4. Combined topical patch

To prevent further development of antibacterial resistance it is recommended that fluoroquinolones be reserved for treatment of: 1. Urinary tract infections in young women 2. Upper respiratory infections in adults 3. Skin and soft tissue infections in adults 4. Community-acquired pneumonia in patients with comorbidities

4. Community-acquired pneumonia in patients with comorbidities

Long-term monitoring of patients who are taking carbamazepine includes: 1. Routine troponin levels to assess for cardiac damage 2. Annual eye examinations to assess for cataract development 3. Monthly pregnancy tests for all women of childbearing age 4. Complete blood count every 3 to 4 months

4. Complete blood count every 3 to 4 months

As drugs near the end of their patent, pharmaceutical companies may apply for the drug to change to over-the-counter status in order to: 1. Get a new patent for the over-the-counter form of the drug 2. Lower the costs because most prescription benefit plans do not cover generics 3. Market the drug to a whole new population, as they are able to market to patients instead of just providers 4. Continue to make large profits from their blockbuster brand-name drug

4. Continue to make large profits from their blockbuster brand-name drug

7. When the costs of a specific treatment or intervention are calculated and then compared with the dollar value of the benefit received it is referred to as: 1. Cost-minimization analysis 2. Cost-of-illness analysis 3. Cost-effectiveness analysis 4. Cost-benefit analysis

4. Cost-benefit analysis

Sharlene is a 65-year-old patient who has been on a lipid-lowering diet and using plant sterol margarine daily for the past 3 months. Her LDL is 135 mg/dL. An appropriate treatment for her would be: 1. A statin 2. Niacin 3. A fibric acid derivative 4. Determined by her risk factors

4. Determined by her risk factors

Beta blockers are prescribed for diabetics with caution because of their ability to produce hypoglycemia and block the common symptoms of it. Which of the following symptoms of hypoglycemia is not blocked by these drugs and so can be used to warn diabetics of possible decreased blood glucose? 1. Dizziness 2. Increased heart rate 3. Nervousness and shakiness 4. Diaphoresis

4. Diaphoresis

Patients who need to remain alert are taught to avoid which drug due to its antimuscarinic effects? 1. Levothyroxine 2. Prilosec 3. Dulcolax 4. Diphenhydramine

4. Diphenhydramine

According to the U.S. Office of Minority Health, poor health outcomes among African Americans are attributed to: 1. The belief among African Americans that prayer is more powerful than drugs 2. Poor compliance on the part of the African American patient 3. The genetic predisposition for illness found among African Americans 4. Discrimination, cultural barriers, and lack of access to health care

4. Discrimination, cultural barriers, and lack of access to health care

If not chosen as the first drug in hypertension treatment, which drug class should be added as a second step because it will enhance the effects of most other agents? 1. ACE inhibitors 2. Beta blockers 3. Calcium channel blockers 4. Diuretics

4. Diuretics

Stage C patients usually require a combination of three to four drugs to manage their heart failure. In addition to ACE inhibitors and beta blockers, diuretics may be added. Which of the following statements about diuretics is NOT true? 1. Diuretics reduce preload associated with fluid retention. 2. Diuretics can be used earlier than stage C when the goal is control of hypertension. 3. Diuretics may produce problems with electrolyte imbalances and abnormal glucose and lipid metabolism. 4. Diuretics from the potassium-sparing class should be used when using an angiotensin receptor blocker (ARB).

4. Diuretics from the potassium-sparing class should be used when using an angiotensin receptor blocker (ARB).

Which of the following is true about procainamide and its dosing schedule? 1. It produces bradycardia and should be used cautiously in patients with cardiac conditions that a slower heart rate might worsen. 2. Gastrointestinal adverse effects are common so the drug should be taken with food. 3. Adherence can be improved by using a sustained release formulation that can be given once daily. 4. Doses of this drug should be taken evenly spaced around the clock to keep an even blood level.

4. Doses of this drug should be taken evenly spaced around the clock to keep an even blood level.

An agonist activates a receptor and stimulates a response. When given frequently over time, the body may: 1. Upregulate the total number of receptors 2. Block the receptor with a partial agonist 3. Alter the drug's metabolism 4. Downregulate the numbers of that specific receptor

4. Downregulate the numbers of that specific receptor

Cecilia presents with depression associated with complaints of fatigue, sleeping all the time, and lack of motivation. An appropriate initial antidepressant for her would be: 1. Fluoxetine (Prozac) 2. Paroxetine (Paxil) 3. Amitriptyline (Elavil) 4. Duloxetine (Cymbalta)

4. Duloxetine (Cymbalta)

Which of the following adverse effects may occur due to a dihydropyridine-type calcium channel blocker? 1. Bradycardia 2. Hepatic impairment 3. Increased contractility 4. Edema of the hands and feet

4. Edema of the hands and feet

There is a narrow margin between first appearance of adverse reaction to AChE inhibitors and serious toxic effects. Adverse reactions that require immediate action include: 1. Dizziness and headache 2. Nausea 3. Decreased salivation 4. Fasciculations of voluntary muscles

4. Fasciculations of voluntary muscles

Treatment costs are important for patients with hypertension. Which of the following statements about cost is NOT true? 1. Hypertension is a chronic disease where patients may be taking drugs for a long time. 2. Most patients will require more than one drug to treat the hypertension. 3. The cost includes the price of any routine or special laboratory tests that a specific drug may require. 4. Few antihypertensive drugs come in generic formulations.

4. Few antihypertensive drugs come in generic formulations.

Levetiracetam has known drug interactions with: 1. Combined oral contraceptives 2. Carbamazepine 3. Warfarin 4. Few, if any, drugs

4. Few, if any, drugs

Treatment of PMDD that affects all or most of the symptoms includes: 1. Tryptophan up to 6 g/d 2. Vitamin E 200-400 mg/d 3. Evening primrose oil 500 mg/d 4. Fluoxetine 20 mg/d

4. Fluoxetine 20 mg/d

Dulcea has type 2 diabetes and a high triglyceride level. She has gemfibrozil prescribed to treat her hypertriglyceridemia. A history of which of the following might contraindicate the use of this drug? 1. Reactive airway disease/asthma 2. Inflammatory bowel disease 3. Allergy to aspirin 4. Gallbladder disease

4. Gallbladder disease

Many disorders require multiple drugs to treat them. The more complex the drug regimen, the less likely the patient will adhere to it. Which of the following interventions will NOT improve adherence? 1. Have the patient purchase a pill container with compartments for daily or multiple times-per-day dosing. 2. Match the clinic appointment to the next time the drug is to be refilled. 3. Write prescriptions for new drugs with shorter times between refills. 4. Give the patient a clear drug schedule that the provider devises to fit the characteristic of the drug.

4. Give the patient a clear drug schedule that the provider devises to fit the characteristic of the drug.

The tricyclic antidepressants should be prescribed cautiously in patients with: 1. Eczema 2. Asthma 3. Diabetes 4. Heart disease

4. Heart disease

The rationale for prescribing calcium blockers for angina can be based on the need for: 1. Increased inotropic effect in the heart 2. Increasing peripheral perfusion 3. Keeping heart rates high enough to ensure perfusion of coronary arteries 4. Help with rate control

4. Help with rate control

Rod, age 68, has hearing difficulty. Which of the following would NOT be helpful in assuring that he understands teaching about his drug? 1. Stand facing him and speak slowly and clearly. 2. Speak in low tones or find a provider who has a lower voice. 3. Write down the instructions as well as speaking them. 4. If he reads lips, exaggerate lips movements when pronouncing the vowel sounds.

4. If he reads lips, exaggerate lips movements when pronouncing the vowel sounds.

Factors that place a patient at risk of developing an antimicrobial-resistant organism include: 1. Age over 50 years 2. School attendance 3. Travel within the U.S. 4. Inappropriate use of antimicrobials

4. Inappropriate use of antimicrobials

Hot flashes are often a concern during menopause. Which of the following may help in reducing them? 1. Drink one caffeinated liquid per day 2. Take progesterone supplementation 3. Exercise 20-40 minutes/day 4. Increase intake of carrots, yams, and soy products

4. Increase intake of carrots, yams, and soy products

Which of the following diagnostic studies would NOT indicate a problem related to a reductase inhibitor? 1. Elevated serum transaminase 2. Increased serum creatinine 3. Elevated creatinine kinase 4. Increased white blood cell counts

4. Increased white blood cell counts

Jack, age 8, has attention deficit disorder (ADD) and is prescribed methylphenidate (Ritalin). He and his parents should be educated about the side effects of methylphenidate, which are: 1. Slurred speech and insomnia 2. Bradycardia and confusion 3. Dizziness and orthostatic hypotension 4. Insomnia and decreased appetite

4. Insomnia and decreased appetite

Passive monitoring of drug effectiveness includes: 1. Therapeutic drug levels 2. Adding or subtracting medications from the treatment regimen 3. Ongoing provider visits 4. Instructing the patient to report if the drug is not effective

4. Instructing the patient to report if the drug is not effective

If a woman presents with recurrent vulvovaginal candidiasis she may be treated with: 1. Weekly intravaginal butoconazole for 3 months 2. Fluconazole 150 mg PO daily x 7 doses then monthly for 6 months 3. Weekly fluconazole 150 mg PO x 6 months 4. Intravaginal tioconazole x 14 days

4. Intravaginal tioconazole x 14 days

Beta blockers have favorable effects on survival and disease progression in heart failure. Treatment should be initiated when the: 1. Symptoms are severe 2. Patient has not responded to other therapies 3. Patient has concurrent hypertension 4. Left ventricular dysfunction is diagnosed

4. Left ventricular dysfunction is diagnosed

Indirect costs associated with drug therapy include: 1. The cost of diagnostic tests to monitor therapeutic levels 2. Health-care provider time to prescribe and educate the patient 3. Child-care expenses incurred while receiving therapy 4. Loss of wages while undergoing drug therapy

4. Loss of wages while undergoing drug therapy

Patients who are prescribed exogenous androgens need to be warned that decreased libido: 1. Is an unusual side effect of androgens and should be reported to the provider 2. Is treated with increased doses of androgens, so the patient should let the provider know if he is having problems 3. May be a sign of early prostate cancer and he should make an appointment for a prostate screening exam 4. May occur with androgen therapy

4. May occur with androgen therapy

Which of the following holds true for the pharmacokinetics of women? 1. Gastric emptying is faster than that of men. 2. Organ blood flow is the same as that of men. 3. Evidence is strong concerning renal differences in elimination. 4. Medications that involve binding globulins are impacted by estrogen levels.

4. Medications that involve binding globulins are impacted by estrogen levels.

Patients with iron deficiency will develop: 1. Hemolytic anemia 2. Megaloblastic anemia 3. Macrocytic-hypochromic anemia 4. Microcytic-hypochromic anemia

4. Microcytic-hypochromic anemia

Larry has heart failure, which is being treated with digoxin because it exhibits: 1. Negative inotropism 2. Positive chronotropism 3. Both 1 and 2 4. Neither 1 nor 2

4. Neither 1 nor 2

Phase I oxidative-reductive processes of drug metabolism require certain nutritional elements. Which of the following would reduce or inhibit this process? 1. Protein malnutrition 2. Iron-deficiency anemia 3. Both 1 and 2 4. Neither 1 nor 2

4. Neither 1 nor 2

Varenicline (Chantix) may be prescribed for tobacco cessation. Instructions to the patient who is starting varenicline include: 1. The maximum time varenicline can be used is 12 weeks. 2. Nausea is a sign of varenicline toxicity and should be reported to the provider. 3. The starting regimen for varenicline is start taking 1 mg twice a day a week before the quit date. 4. Neuropsychiatric symptoms may occur.

4. Neuropsychiatric symptoms may occur.

Isosorbide dinitrate is a long-acting nitrate given twice daily. The schedule for administration is 7 a.m. and 2 p.m. because: 1. Long-acting forms have a higher risk for toxicity. 2. Orthostatic hypotension is a common adverse effect. 3. It must be taken with milk or food. 4. Nitrate tolerance can develop.

4. Nitrate tolerance can develop.

Providers have a responsibility for determining the best plan of care, but patients also have responsibilities. Patients the provider can be assured will carry through on these responsibilities include those who: 1. Are well-educated and affluent 2. Have chronic conditions 3. Self-monitor drug effects on their symptoms 4. None of the above guarantee adherence

4. None of the above guarantee adherence

The most appropriate smoking cessation prescription for pregnant women is: 1. A nicotine replacement patch at the lowest dose available 2. Bupropion (Zyban) 3. Varenicline (Chantix) 4. Nonpharmacologic measures

4. Nonpharmacologic measures

Vitamin B12 deficiency may lead to: 1. Hair loss 2. Insomnia 3. Dry scales on the scalp 4. Numbness and tingling of the hands

4. Numbness and tingling of the hands

There is enough preliminary evidence to recommend that children with autism receive which supplemental nutrient? 1. Vitamin B1 (thiamine) 2. Vitamin B2 (riboflavin) 3. Calcium 4. Omega-3 fatty acids

4. Omega-3 fatty acids

Josie is a 5-year-old patient who presents to the clinic with a 48-hour history of nausea, vomiting, and some diarrhea. She is unable to keep fluids down and her weight is 4 pounds less than her last recorded weight. Besides IV fluids, her exam warrants the use of an antinausea medication. Which of the following would be the appropriate drug to order for Josie? 1. Prochlorperazine (Compazine) 2. Meclizine (Antivert) 3. Promethazine (Phenergan) 4. Ondansetron (Zofran)

4. Ondansetron (Zofran)

Methylnaltrexone is used to treat constipation in: 1. Patients with functional constipation 2. Patients with irritable bowel syndrome-associated constipation 3. Children with encopresis 4. Opioid-associated constipation

4. Opioid-associated constipation

An Investigational New Drug is filed with the U.S. Food and Drug Administration: 1. When the manufacturer has completed phase III trials 2. When a new drug is discovered 3. Prior to animal testing of any new drug entity 4. Prior to human testing of any new drug entity

4. Prior to human testing of any new drug entity

What dermatological issue is linked to Amiodarone use? 1. Increased risk of basal cell carcinoma 2. Flare up of any prior psoriasis problems 3. Development of plantar warts 4. Progressive change of skin tone toward a blue spectrum

4. Progressive change of skin tone toward a blue spectrum

When treating patients using the "Step-Down" approach the patient with gastroesophageal reflux disease is started on _______ first. 1. Antacids 2. Histamine2 receptor antagonists 3. Prokinetics 4. Proton pump inhibitors

4. Proton pump inhibitors

Selma, who is overweight, recently started taking topiramate for seizures and at her follow-up visit you note she has lost 3 kg. The appropriate action would be: 1. Tell her to increase her caloric intake to counter the effects of the topiramate. 2. Consult with a neurologist, as this is not a common adverse effect of topiramate. 3. Decrease her dose of topiramate. 4. Reassure her that this is a normal side effect of topiramate and continue to monitor her weight.

4. Reassure her that this is a normal side effect of topiramate and continue to monitor her weight.

Monitoring of a patient on gabapentin to treat seizures includes: 1. Routine therapeutic drug levels every 3 to 4 months 2. Assessing for dermatologic reactions, including Steven's Johnson 3. Routine serum electrolytes, especially in hot weather 4. Recording seizure frequency, duration, and severity

4. Recording seizure frequency, duration, and severity

Charlie is a 65-year-old male who has been diagnosed with hypertension and benign prostatic hyperplasia. Doxazosin has been chosen to treat his hypertension because it: 1. Increases peripheral vasoconstriction 2. Decreases detrusor muscle contractility 3. Lowers supine blood pressure more than standing pressure 4. Relaxes smooth muscle in the bladder neck

4. Relaxes smooth muscle in the bladder neck

Asians from Eastern Asia are known to be fast acetylators. Fast acetylators: 1. Require acetylization in order to metabolize drugs 2. Are unable to tolerate higher doses of some drugs that require acetylization 3. May have a toxic reaction to drugs that require acetylization 4. Require higher doses of drugs metabolized by acetylization to achieve efficacy

4. Require higher doses of drugs metabolized by acetylization to achieve efficacy

The Combat Methamphetamine Epidemic Act, which is part of the 2006 U.S. Patriot Act: 1. Requires all providers to screen their patients for methamphetamine use 2. Restricts the prescribing of amphetamines to U.S. citizens 3. Requires a prescription be written for all methamphetamine precursors in all states 4. Restricts the sales of drugs that contain methamphetamine precursors, including a daily and 30-day limit on sales

4. Restricts the sales of drugs that contain methamphetamine precursors, including a daily and 30-day limit on sales

Women who are taking an oral contraceptive containing the progesterone drospirenone may require monitoring of: 1. Hemoglobin 2. Serum calcium 3. White blood count 4. Serum potassium

4. Serum potassium

A comprehensive assessment of a patient should be holistic when trying to determine competence in drug administration. Which of the following factors would the NP omit from this type of assessment? 1. Financial status 2. Mobility 3. Social support 4. Sexual practices

4. Sexual practices

Men who use transdermal testosterone gel (AndroGel) should be advised to avoid: 1. Washing their hands after applying the gel 2. Wearing occlusive clothing while using the gel 3. Exposure to estrogens while using the gel 4. Skin-to-skin contact with pregnant women while using the gel

4. Skin-to-skin contact with pregnant women while using the gel

Jamie has glucose-6-phosphate dehydrogenase deficiency (G6PD) and requires an antibiotic. Which class of antibiotics should be avoided in this patient? 1. Penicillins 2. Macrolides 3. Cephalosporins 4. Sulfonamides

4. Sulfonamides

To prevent life-threatening events from rapid withdrawal of a beta blocker: 1. The dosage interval should be increased by 1 hour each day. 2. An alpha blocker should be added to the treatment regimen before withdrawal. 3. The dosage should be tapered over a period of weeks. 4. The dosage should be decreased by one-half every 4 days.

4. The dosage should be decreased by one-half every 4 days.

Which of the following is true concerning lesbian health concerns? 1. They cannot contract an STI from another woman. 2. Pap smears are not required to screen for cervical cancer. 3. Lesbian women have a tendency to be frequent clinic visitors. 4. The health risks associated with smoking, alcohol, and depression are higher than in the heterosexual population.

4. The health risks associated with smoking, alcohol, and depression are higher than in the heterosexual population.

The laboratory monitoring required when a patient is on a selective serotonin reuptake inhibitor is: 1. Complete blood count every 3 to 4 months 2. Therapeutic blood levels every 6 months after a steady state is achieved 3. Blood glucose every 3 to 4 months 4. There is no laboratory monitoring required

4. There is no laboratory monitoring required

Common mistakes practitioners make in treating anxiety disorders include: 1. Switching medications after an 8- to 12-week trial 2. Maximizing dosing of antianxiety medications 3. Encouraging exercise and relaxation therapy before starting medication 4. Thinking a partial response to medication is acceptable

4. Thinking a partial response to medication is acceptable

Class I recommendations for stage A heart failure include: 1. Aerobic exercise within tolerance levels to prevent the development of heart failure 2. Reduction of sodium intake to less than 2,000 mg/day to prevent fluid retention 3. Beta blockers for all patients regardless of cardiac history 4. Treatment of thyroid disorders, especially if they are associated with tachyarrhythmias

4. Treatment of thyroid disorders, especially if they are associated with tachyarrhythmias

Cynthia is taking valproate (Depakote) for seizures and would like to get pregnant. What advice would you give her? 1. Valproate is safe during all trimesters of pregnancy. 2. She can get pregnant while taking valproate, but she should take adequate folic acid. 3. Valproate is not safe at any time during pregnancy. 4. Valproate is a known teratogen, but may be taken after the first trimester if necessary.

4. Valproate is a known teratogen, but may be taken after the first trimester if necessary.

When are statins traditionally ordered to be taken? 1. At bedtime 2. At noon 3. At breakfast 4. With the evening meal

4. With the evening meal

Factors that explain and predict medication adherence include: 1. Social 2. Financial 3. Health system 4. All of the above

4.All of the above

Sarah developed a rash after using a topical medication. This is a type __ allergic drug reaction. 1. I 2. II 3. III 4. IV

4.IV

All drugs continue to act in the body until they are changed or excreted. The ability of the body to excrete drugs via the renal system would be increased by: 1. Reduced circulation and perfusion of the kidney 2. Chronic renal disease 3. Competition for a transport site by another drug 4. Unbinding a nonvolatile drug from plasma proteins

4.Unbinding a nonvolatile drug from plasma proteins

Androgens are indicated for: 1. Symptomatic treatment for male deficiency 2. Female libido, endometriosis, and postmenopausal symptoms 3. Increased muscle mass 4. Symptomatic treatment in both sexes for cancer and HIV 5. 1, 2, and 4 6. All of the above

5. 1, 2, and 4

Nurse practitioner care may thrive under health-care reform because of: 1. The demonstrated ability of nurse practitioners to control costs and improve patient outcomes. 2. The fact that nurse practitioners will be able to practice independently 3. The fact that nurse practitioners will have full reimbursement under health-care reform 4. The ability to shift accountability for Medicaid to the state level

The demonstrated ability of nurse practitioners to control costs and improve patient outcomes.


Kaugnay na mga set ng pag-aaral

Spanish 2, ¿Qué? and ¿cuál?, Lesson 9.3

View Set

para practicar su pronunciación de "r" y "rr" en español

View Set

A Beka 4th Grade History Quiz 27

View Set

Environmental Science Chapter 13

View Set

Web Design - Unit 1: The History of the Internet

View Set